An interesting question.

A namesake, Richard Christie, has asked me an interesting question at Open Parachute and as I decided to give a somewhat extensive answer I am posting it here. Richard first asked my opinion on the issue of the Catholic Church actively suppressing various lines of scientific enquiry. I asked him to be more specific as I knew of no case where the Church had succeeded in suppressing a scientific line of enquiry. Richard quite correctly pointed out that actively suppressing and succeeding in suppressing are actually two different things. He then suggested the Church’s inclusion of Kepler in the Librorum Prohibitorum, commonly called the Index, as an example of the active suppression of various scientific activities.

The whole question of the Index and the inclusion of scientific books, in particular books on heliocentrism, in it is both a very complex one and a very emotional one that is usually dealt with in a very superficial manner in discussions about science and religion and although I can’t even attempt to give a complete answer in a blog post I will try to at least touch upon many of the factors involved.

The first thing that needs to be considered is the question of the censorship of books in general in the 16th and 17th centuries something that is almost always ignored in discussions of the Index.   The Catholic Church was by no means the only authority that tried to control the printing and distribution of books, in fact for most of the 16th and 17th century nearly all political authorities, and it should be remembered that The Vatican was a major political force and not just a religious one, tried to exercise this control through systems of licensing and bans. The printed word was regarded as dangerous and governments throughout Europe tried desperately to control it. I hadn’t realised until I started thinking about writing this post how similar the situation in those days with the newly invented medium, the printed book, is to our current situation with politicians of every hue trying to find ways to control and censor the Internet. I wont investigate the parallels further but I think the informed readers will see them for themselves. These attempts at control usually took the form of licensing, everyone wishing to print and publish a book, pamphlet or what ever being required to first obtain permission in the form of a license from the authorities. Anything printed without a license was automatically banned. Anything regarded as seditious for whatever reasons the political winds of the day decreed was potentially dangerous was denied a license. This would have meant that in a perfect world the authorities would have retained complete control over everything that appeared in print. This as we shall soon see was far from the truth.

In academic discussions of the law, the results of legislation are divided into good and bad laws. This division has little to do with the ethical or moral contents of a piece of legislation, although that also gets discussed, but rather with technical legal aspects of the law under discussion. A central point in such discussion is enforceability, a law that is difficult or almost impossible to enforce is per definition a bad law. In terms of enforceability the laws designed to ensure control of the printed word in the Early Modern Period were bad laws. Those wishing to publish seditious material, or anything that might be considered so, did not apply for a license in the first place but set up an underground press and published illegally. Of course people being what they are the attraction of the forbidden meant that the fruits of the underground presses enjoyed a massive popularity making something of a mockery of the attempt to censor them in the first place. If things got too hot for the operator of an underground press he just simply upped sticks and moved to another country where his products were not regarded as seditious. Often one government would actively encourage the seditious presses of a rival country for political reasons. The puritan agitators in England in the early part of the 17th century in the period before The English Revolution (or Civil War) moved their presses to the newly created United Provinces of the Netherlands whose Calvinist Government was sympathetic towards their cause. On the whole attempts to control or censor the printed word were a farce and totally ineffective, which however does not mean that those who got caught were not severely punished for their activities. Things were not any different for books or pamphlets placed upon the Church’s Index.

Having taken a brief look at the general background let us now turn to the attempt by the Catholic Church to control through censorship the literature on heliocentrism, I have generalised the original question concerning Kepler; in order to do this we need to be clear what exactly happened in 1616 and why. First of all what happened in 1616 actually had very little to do with science, this might seem like a rather strange statement given the fact that the books that were placed on the Index were books propagating heliocentrism but it is none the less true. What was a stake here was theology and not science and not even a conflict between theology and science but the question, who had the right to determine theological questions.

First of all one has to remember that at the beginning of the 17th century almost all educated people believed in the literal truth of the Bible, something that was not without problems as the Bible is often vague, confused and even contradictory, which means one first has to determine what the truth of the Bible is. The Catholic Church as an organisation had in its existence claimed the exclusive right to carry out this interpretation and its political power actually lay in this exclusive right. In this the Catholic Church does not differ substantially from other religious organisation and the intertwining of religion and politics to exercise control over the common folk is almost universal. Now the Bible often stands in conflict with reality and it is the job of theologians to sort out these conflicts. A theologian is not able to change reality so he must find a way to interpret the Bible that removes the conflict and by the beginning of the 17th century the Church’s theologians had become quite skilled at dissolving such conflicts. The relatively new heliocentric hypothesis potentially presented one such conflict as various passages in the Bible seemed to support a geocentric universe, which was not surprising as all the empirical evidence available up till that time also supported a geocentric universe. If the evidence for a heliocentric model had become stronger, which it did with the telescopic discoveries made between 1610 and 1613, then almost certainly the Church’s theologians would in the fullness of time have found a way to interpret the troublesome Bible passages in order to remove the conflict. As Galileo had been told by several of his influential Church friends, including Barberini the future pope, in 1611, as he was basking in the glory of his Sidereus Nuncius, urging him to be patient. Both the writings of Augustus Augustine and Thomas the two principal touch stones for Catholic theology provided possibilities for such interpretations. In fact Thomas had together with his teacher Albertus Magnus achieved just such a conflict solution with the scientific writings of Aristotle. However two people forced the issue and precipitated a minor theological crisis in the second decade of the 17th century, Galileo Galilei and Paolo Antonio Foscarini a Carmelite friar. The crime the these two men committed in the Church’s eyes was not that they propagated heliocentrism, which they did, but that they told the Church how to interpret the Bible and that was definitely a no, no.

I have already explained that the Church’s power was centred on their claim to the exclusive right to interpret the Bible, so Galileo and Foscarini telling them how to do so would have been bad enough at any time but you have to remember that they were in the middle of the Counter Reformation and on the eve of the Thirty Years War. One of the central claims of the Reformation had been that every man was entitled to read and interpret the Bible for himself without the authority of the Church, a claim that threatened to undermine the central power of the Catholic Church, so anybody coming along and trying to tell the Church how to interpret the Bible on any theme was treading on very dangerous territory especially as the Church was also a major absolutist political power in Italy at this time. Things took the inevitable course Galileo and Foscarini both got a rap on the knuckles and books claiming that the heliocentric hypothesis was true were placed on the Index, including Kepler’s.

This situation raises two immediate questions, what did the Church actually ban and why did they think it necessary to do so. What the Church banned was the heliocentric theory that is the claim that the universe is in reality heliocentric so all books making this claim were to be placed on the Index. The reasons for this ban are not so straightforward as they at first might appear. The potential conflict between heliocentricity and some passages in the Bible had existed since the Bible was collated in the 4th century, as Ptolemaeus had already discussed and rejected the heliocentric hypothesis in his Syntaxis Mathematiké in the 2nd century. Nobody had been really bothered by the potential conflict until Galileo and Foscarini had made it into a real conflict by suggesting a theological solution thus creating a real problem for the Church, which had three possible solutions. Firstly, they could take the course that would seem most logical to the modern mind and simply reinterpret the Bible to remove the conflict, this was of course, seen through the eyes of the Church in the 17th century, impossible. If they did so they would be admitting that Galileo and Foscarini were right and in admitting that a mere mathematicus could interpret the Bible giving up their exclusive right to do so, the basis of their power. They could of course ignore the whole thing and do nothing, again an impossible situation for the Church because then it would appear that they were incapable of solving the conflict i.e. of correctly interpreting the Bible. This process of reasoning led to the only possible conclusion that the Bible was right and the heliocentric theory was wrong and therefore may not be propagated and any books that did so must be banned. In his unconsidered and over hasty actions Galileo had forced the Church to ban the heliocentric theory.

Those who have been paying attention may have noticed that I have alternated between two different terms the heliocentric theory and the heliocentric hypothesis. This is neither sloppiness on my part nor an attempt to avoid repetition of words for reasons of style. This is very deliberate because they are actually two substantially different terms. The heliocentric hypothesis says that heliocentricity offers a possible model to explain the observed motion of the planets; it says nothing about the truth-value of this model. The heliocentric theory says that the universe is in reality heliocentric. In 1616 the Church banned the heliocentric theory but not the hypothesis. This might at first seem like splitting hairs but in reality it is a very important distinction. Astronomers were completely free to go on discussing and researching the possibility of heliocentricity but until they produced actual proof that the universe is indeed heliocentric they were not allowed to claim that it was. So in reality the Church was here not even attempting to actively suppress a line of scientific activity. As a side note it should be pointed out from an epistemological standpoint the Church was right to deny the correctness of the heliocentric theory at that time, which does not however excuse their primitive attempt to ban it.

It might be argued that the Church created this obvious loophole out of ignorance or by mistake but the historical evidence clearly shows this was not the case. Firstly the much maligned Roberto Bellermino wrote in his so-called Foscarini letter that should the heliocentric hypothesis be proved to be correct then the Church would have to re-interpret the Bible, showing that he was well aware of the difference between hypothesis and theory. Secondly, contrary to popular opinion, the main heliocentric text, Copernicus’ De revolutionibus, was not placed on the Index but only referred to the Index until corrected. This meant that all the passages claiming the reality of heliocentricity were to be removed. This was actually carried out and the De revolutionibus, with surprisingly few minor changes was released again for general consumption in 1622.

So what were the consequences of the Church’s ban on books containing the heliocentric theory, all in all very few. The Church managed to embarrass itself for the next 400 years and the astronomers got on with doing what they had been doing before Galileo tried his hand at theology. Kepler was at first worried about the consequences of having his book placed on the Index but his friends pointed out to him that this would actually make them more attractive, forbidden fruit. All of the astronomy books on the Index were actually readily available to scholars, all the ban meant was that they could not find them on open shelves but had to apply to read them and they would be duly delivered from the poison cabinet. Outside of Italy nearly all scholars just simply ignored the ban and Catholic astronomers didn’t even bother to correct their copies of De revolutionibus. One notable exception was René Descartes who held back the publication of one of his books on heliocentricity out of respect for his Jesuit teachers. This fact has a certain level of irony because as Descartes was exercising restraint the Jesuit astronomer Johann Adam Schall von Bell was teaching Copernican astronomy in China. Probably the major source through which scholars learnt about the Copernican theory in the middle of the 17th century was the textbook of Pierre Gassendi the professor for astronomy and mathematics at the university in Paris, like Descartes a product of the Jesuit education system. Gassendi’s book of course taught heliocentrism as a hypothesis. Within Italy things were slightly more critical and Italian astronomers trod somewhat more carefully. Their books on heliocentricity presented it as a hypothesis and included the information that the Church had ruled that it was not true. Two leading Jesuit astronomers Athanasius Kircher and Giovanni Riccioli were both reported to the Inquisition for sailing too close to the wind on the heliocentric theory but both were eventually cleared of suspicion.

In the end the Church took a long time to back itself out of the cul de sac into which Galileo and Foscarini had steered it but the consequences for the development of astronomy of placing pro-heliocentric books on the Index were negligible and it has even been argued that Bellarmino’s statement that a proof of heliocentricity would force the Church to re-interpret the Bible actually spurred Catholic and in particular Jesuit astronomers on to find the necessary proof.

I will close with a personal statement that ought to be totally superfluous but there are some people underway in the Internet who think that when one discusses heliocentricity and the Index if one doesn’t scream Church bad, Church evil whilst beating ones breast in best King Kong manner then one condones or even supports the Church’s censorship. For the record I vehemently oppose all forms of censorship not matter who exercises them.

136 Comments

Filed under History of Astronomy, History of science, Renaissance Science

136 responses to “An interesting question.

  1. Pingback: An interesting question. | Whewell's Ghost

  2. Thanks Thony. A most useful summary which I shall do my best to propagate (and sincere apologies for previously being incapable of spelling your name right. This is probably because I am an evil Catholic myself and therefore quite unable to manage even the simplest tasks requiring the use of reason).

    • Don’t worry James in some circles I’m an evil Jesuit lover who does unspeakable things to small children whilst lying about being an atheist. One of my crimes is that I defended your opinion of Galileo’s Dialogo!

      BTW glad you enjoyed my thoughts on the subject.

    • isomorphismes

      That’s no more appropriate than for the old Church to make scientists swear their adherence to Doctrine. Facts are facts, no matter who they’re stated by.

  3. Markk

    So basically the church did try to suppress lines of scientific communication, thus lines of inquiry, when they conflicted with their power to interpret the bible. I can’t see how Galileo and Foscarini “drove” the church anywhere. They were used as tools by others if anything.

    • While there were certainly influences external to these figures, both men (but especially Galileo) were quite in control of what they were doing. As Thony said, the issue wasn’t the science, per se, but the attempt made by Galileo in particular to claim that the bible didn’t conflict with the heliocentric model.

      The fine line came down to what constituted absolute truth – astronomers were permitted to propose models that gave a metaphorical interpretation of nature. In fact, the church had no real problem with the teaching of heliocentrism, so long as no attempt was made to say that was how the universe really, truly was. After all, the bible was absolute truth, such models were but pale attempts to describe nature by comparison. But when Galileo attempted to draw a connection between the two, he risked stating that the scientific models were on equal footing with what the bible said, which risked subverting Church authority on how the bible should be interpreted. As Thony said, it was less an issue of stamping down on science as a concept and more on retaining the authority of interpretation. Galileo might have been skirting the line drawn for him, but by all accounts he seemed to do so with a desire to overcome his personal dissonance and marry the authority of science with the authority of the bible.

  4. Excellent post. Definitely one to keep on file and to propagate to those who can only indulge in the King Kong chest-beating reaction to this subject.

    I have the misfortunte of being a rare breed: an atheist with an interest in the *objective* study of the history of science. As a result, if I try to present any version of that history that tries to correct the post-Enlightment/Draper-White myths (or even one free of modern value judgements and outrage) I get accused of being a closet Christian apologist.

    In fact, I recently asked the Australian Atheist Foundation to send me a laminated copy of my subscriber’s certificate so I can scan it to send to those who accuse me of being a secret employee of the Vatican for stating heresies such as “the Medieval Church didn’t burn any scientists” or “the Galileo Affair was not quite what you’ve been told it was” and other outrageous things.

    • Tim it’s nice to know that I’m not alone, as we seem to share a common interest in historical accuracy. If you want to know how at least one ‘atheist’ views me read my answer to James above.

    • Ye Olde Statistician

      Here is what Aquinas had to say on the matter:
      The suppositions that these astronomers have invented need not necessarily be true; for perhaps the phenomena of the stars are explicable on some other plan not yet discovered by men
      De coelo, II, lect. 17

      “The theory of eccentrics and epicycles is considered as established, because thereby the sensible appearances of the heavenly movements can be explained; not, however, as if this proof were sufficient, forasmuch as some other theory might explain them.”
      Summa theologica, I, q.32, a.1, ad. 2

      He also said, following Augustine:
      Since Holy Scripture can be explained in a multiplicity of senses, one should adhere to a particular explanation only in such measure as to be ready to abandon it, if it be proved with certainty to be false; lest Holy Scripture be exposed to the ridicule of unbelievers, and obstacles be placed to their believing.
      Summa theologica, Part I, Q. 68, art. 1

      The problem was, the scientific consensus was pretty much against heliocentrism, so Galileo’s attempt to re-interpret Scripture seemed suspicious. That the established science backed the Church Fathers made matters seem even more securely determined. So it’s not so much that they were clinging to the pagan Greek cosmology because “the Bible sez.” The Roman and Orthodox churches were not into naive-literalism as such (though the Romans were being defensive about it in the 17th cent. in the face of the Protestant critique,) but rather that the science seemed to back geocentrism and had done so for more than two millennia. For a comparison, imagine someone coming along today and saying Relativity is wrong or Darwinian evolution is wrong, and I got this neat new theory. We’d want more than just his say-so.

      • Michael Fugate

        In 1610, what was the evidence that supported geocentrism over heliocentrism?
        Tradition?

      • We’d want more than just his say-so.

        We would indeeed

      • Tim O'Neill

        “In 1610, what was the evidence that supported geocentrism over heliocentrism?”

        The same evidence that had convinced Aristotle to be a geocentrist in 300 BC or Ptolemy to be one in 150 AD. Before Newton revolutionised our understanding of physics it was hard to see how the earth could be rotating without the consequent inertia making everything fly off into space. And before telescopes were precise enough, there was no way to observe the stellar parallax that would show that the earth was orbiting the sun.

        In 1610 both of these arguments were powerful objections to heliocentrism that Galileo could not answer. It’s interesting that the Church and all the scientists who dismissed Galileo used the same arguments used by the Greeks and Romans – the same Greeks and Romans lauded as being beacons of rationalism by the historical semi-literates over at “Butterflies and Wheels”. See anything wrong with this picture?

      • Michael Fugate

        Which do you think weighed more heavily in response to Copernicus and Galileo – the inertia argument or the biblical argument? The Greeks and Romans (well everyone even in the 17th c.) were relying on bad data or no data. Mostly what they had were assumptions and assertions on the size and weight of the planets, the size of the universe, the movement of physical bodies, etc. We know heliocentric ideas were discussed in Greek circles and that heliocentrism solved some problems and created others. To claim that the evidence supported geocentrism is shaky at best – to claim that geocentrism became the dominant model is obviously true, but the reasons for its support may not be strongly evidence-based. I have neither defended the thesis that Christians destroyed Greek and Roman “science” nor promoted the view that Christianity was responsible for modern science. I think religion is largely irrelevant to the issue.

      • Michael see next post

      • isomorphismes

        This may be completely off, but Aquinas’ speculations sound like certain modern views on quantum mechanics. “These correctly predict the movements but we will say no more.” So I wonder if we don’t still distinguish between “hypothesis” and “theory” in the 17th century sense defined by Thony above.

      • isomorphismes

        Tim, I don’t catch your point here:

        It’s interesting that the Church and all the scientists who dismissed Galileo used the same arguments used by the Greeks and Romans – the same Greeks and Romans lauded as being beacons of rationalism by the historical semi-literates

        Given the state of measurement their arguments do seem quite rational. As far as the science goes, every discussion I’ve seen from heliocentrists or geocentrists seems reasonable and aware, for example Aquinas’ statement in Summa theologica, Part I, Q. 68, art. 1.

  5. I wouldn’t completely underestimate the importance of censorship in maintaining orthodoxy. The breakdown of the English censorship system in 1641 surely had a role in the outbreak of the Civil Wars, and Robert Darnton has documented how the scurrilous popular press discredited the French monarchy. The phiosophes were more realistic than we tend to be about the genuine dangers that go along with the freedom of the press. When the Danes ended censorship in 1770, the Republic of Letters debated whether this reform was premature. They favored freedom, naturally; but they knew it presented risks to public peace and wondered aloud if Danish society had become sufficiently enlightened to permit press liberty. (Long quotations from this and related debate are reproduced in Franco Venturi’s End of the Old Regime in Europe, 1768-1776, Chapter VIII.)

    The printing revolution created particular problems for science and not simply because the churches policed scientific publications. Just as the advent of the Internet vastly and suddenly increased the sheer bulk of available nonsense, an enormous amount of what appeared in the wake of the invention of moveable type was the sheerest crap. Humanistic texts, let alone scientific texts, formed a relatively small proportion of what emerged, especially in comparison to religious publications, which were the indispensable foundation of the printing industry. One thinks of the Renaissance and Baroque as eras when science advanced; but the case can and has been made that it was these years that saw the triumph of Christianity, which was finally able to penetrate even the dimmest recesses of agrarian Europe thanks in part to a huge program of missionary work made entirely more effective with the aid of cheap prayer books and pamphlets (vernacular bibles for the Protestants). At the same time, vastly popular almanacks and woodcuts spread the reach of popular superstition, while more highfalutin folks now had access to a wide range of formerly hard to acquire esoterica. To keep from drowning in this brown flood, the emerging network of natural philosophers had to develop the beginnings of science’s own system for controlling discourse. Surviving peer review is not the same process as getting an imprimatur from Cardinal Bellarmine but if it is different, it is at least as rigorous. We don’t think of it as a system of censorship because we largely approve of it–I do, anyhow–but it seems to me that that’s exactly what it is. Of course the rules of scientific publication haven’t prevented the sciences from entertaining and accepting a host of ideas that are at least as original as anything that ever arose from the theology business, but the creativity of science, or so I claim, is made possible precisely because it has figured out how to wall itself off from general public discourse and even commonsense. Without insulation, short circuits. Without the thermos bottle, spoiled milk.

    I wonder if it isn’t the case that censorship (broadly defined) was even more important for the advance of science than for the success of the Counter Reformation, which found better tactics. Banning books, as you point out, didn’t work so well for the church, which could never close porous borders or defeat the lure of the forbidden. Burning people at the stake turned out to be less effective in the long run, also. My reading of how the Counter Reformation won where it did win is that the most effective thing a Catholic prince could do to further the faith was to make orthodoxy a prerequisite of lucrative office. Young men may die for their version of the faith, but it’s a more serious matter to forgo a good job and social advancement. Patronage combined with a determined campaign of propaganda and moral terrorism for the masses does the trick. Mutatis mutandis, after all, the same formula is working pretty well for modern political conservatism.

  6. Brian

    Thony, as a rabid gnu, I really liked this.

    This made me wonder:

    Both the writings of Augustus and Thomas the two principal touch stone for Catholic theology provided possibilities for such interpretations.

    Do you mean Augustine?

  7. Could you clarify here – when you talk of “interpreting the Bible”, did the heliocentric theory in itself originally count as interpreting the Bible, or only once Galileo and others started specifically saying such things as “And therefore what must have been meant by the sun stopping is …”? In the former case you really could say that the Church was in conflict with science. Since you say this was not the case I assume that’s not what you mean but it would be helpful to have that clarified.

    Regarding the Church’s options – did they really have no option of saying “Yes, these guys happened to get it right by coincidence, but you only know they got it right because we said it”? Now I imagine doing this repeatedly would be harmful to their authority… did they foresee that they would be stuck doing that if they did it once? Or what about simply saying “Yes, they happened to get it mostly right, but actually…” and then making minor corrections to Galileo’s interpretation (even if they had to invent them for the purpose)?

    • Sniffnoy, the heliocentric hypothesis states that the sun is stationary in the middle of the universe and the earth together with the other planets circle the sun as well as turning once a day on its own axis. The story of Joshua in the Bible states that during the siege of Jericho the sun stood still. If the sun does not move how could it stand still? Viewed on a direct simplistic level the two accounts, heliocentricity and Joshua, are contradictory. In order to avoid the contradiction if the heliocentric hypothesis is true and the Bible is the true word of God one has to re-interpret the passage in the Bible, one can’t of course re-interpret heliocentricity if its true. Such interpretations or re-interpretations is the principal function of theology.

      Here it’s not about Galileo being right or wrong in his theological re-interpretation of the Bible but about the fact that in Catholicism only the Church is allowed to interpret the Bible. Any attempt at that time to adopt Galileo’s interpretations would be to admit that a mere mathematicus is allowed to interpret the Holy word and such an admission was completely and utterly impossible from the Church’s standpoint.

      • isomorphismes

        I don’t understand how that story implies non-heliocentricity. Clearly the Sun moves throughout the sky during most days (hence why it’s worth remarking that the Sun stood still during the siege), including all of the ones we live through. It would seem that making the Sun stand still is a Miracle of God. What am I missing here?

      • In the heliocentric theory the suns movement is only an illusion. To stop the apparent movement of the sun the earth would have to stand still.

      • isomorphismes

        Still — it would seem that that passage refers to a miracle (stopping the Earth, I guess). I mean, the Sun does pass thru the sky every day.

  8. Brian

    Thony, a thought popped into my brain an it is most likely wrong.

    In the early part of the article you state that theologians can’t dictate reality and so must interpret scripture to suit reality. Later you say that

    As a side note it should be pointed out from an epistemological standpoint the Church was right to deny the correctness of the heliocentric theory at that time,

    Isn’t this a case of the church trying to dictate reality? I mean if you say ‘hypothesis as you will, but don’t claim reality’ then that’s not theology but telling all and sundry that you control claims to reality. It seems to be more than adjusting theology to suit reality.

    • Actually Brian it was those who claimed that the heliocentric theory was reality who were doing the unjustified dictating. In 1616 most of the available empirical evidence spoke against a moving earth and therefore against heliocentricity. There existed no scientifically acceptable proof of the truth of heliocentricity so to claim that it was true was epistomologically false.

      The Church in this instance was not, as you put it, telling all and sundry that they controlled claims to reality merely insisting quite correctly that the heliocentric hypothesis was a long way away from being proved and until that proof was delivered it would not accept claims of the truth of heliocentricity.

      Although modern science does not use censorship to achieve its aims fundamentally it does not behave differently. If you wish to claim the truth of a scientific hypothesis then you have to deliver acceptable scientific proof of your claim. If you fail to do so then the scientific community will publicly reject your truth claims and not allow you to publish them in peer reviewed scientific publications.

      • Brian

        Thanks Thony, but the church didn’t just take the epistemologically correct route did it? It didnt’ just say, ‘well, there’s no compelling evidence for heliocentric theory, so therefore nobody is justified in holding it true’. I understood from your post that the church arrogated to itself the right to say what was epistemologically true. If the church, for theological or other reasons didn’t like an outcome, they ruled it not true. Which is dictating reality to others it would seem.

      • Then you have failed to understand my post as that is exactly what I didn’t say. I’m afraid I don’t know what to say now as I don’t know how you could possibly draw the conclusion that you have drawn from my thoughts.

  9. Brian

    I must be very wrong then. I’ll try to explain how I’ve come to this….

    What the Church banned was the heliocentric theory that is the claim that the universe is in reality heliocentric so all books making this claim were to be placed on the Index.

    So the Church decided, for reasons affecting its political power to decree what is not reality. I understand that the church did this because Galileo assumed the truth of heliocentric theory and interpreted the Bible in that light instead of letting the Church do this. The Church having arrogated to themselves the right to interpret the Bible and match it to reality for political reasons (control, power) felt they must reject the interloper’s ideas and not only declare his theological musings false, but also his scientific ( if you could call it science back without being anachronistic ) or natural philosophical claims false. This appears to me to be arrogating not only the right to decide theological, but also physical (science) questions. Now claims about heliocentric theory being false, are epistemic claims and the Church has made them. As you say, their was not the justification to hold heliocentric theory true, but that doesn’t mean it was wrong, but an active hypothesis. It was an overreach to declare it false. But the Church had in shoring up their claims to be the sole interpreter of theology stretched much further and in proscribing a theory in this case, declared themselves in a position to say what was epistemologically true.

    Well, I hope that makes sense. I’m not trying to be a pain. Just trying to get it straight. Thanks.

    • So the Church decided, for reasons affecting its political power to decree what is not reality.

      WRONG The Church said that one was not allowed to claim that heliocentricity is true until it had been proved to be so. The Church didn’t decree reality it forbad other people to do so without proof which is completely different.

      The Church never declared the heliocentric theory to be false they only said that it had not been proved

      As soon as heliocentricity was proved to be true about 1750 the Church lifted its ban on the heliocentric theory.

  10. Whatever its faults, you just can’t claim that 17th Century Catholicism didn’t care about philosophy or science. One of the critical decisions of the Council of Trent, the great meeting that defined Counter Reformation, had been to endorse St. Thomas who had been exquisitely clear on the difference between theology and philosophy. Somewhat anachronistically, we tend to think that the great and novel problem for the church was negotiating the difference between dogma and emerging science, but the Galileo debate was superimposed on a much older struggle to assimilate Greek though, specifically Aristotle, to church teaching. Nobody gives a damn about them, but a long roll call of philosophy professors ran afoul of the Inquisition because they interpreted Aristotle too literally or took him to seriously on issues like the eternity of the world, the immortality of the soul, and the question of whether God could know individuals. Unfortunately, if you get thrown in a dungeon for agreeing too publicly with Averroes, they won’t make a movie about you.

    Meanwhile, Pope Urban wasn’t exactly channeling Karl Popper when he chopped logic with Galileo, but his epistemology is not so far from 20th or 21st Century commonplaces about the status of empirical knowledge. It’s actually not so easy to formulate a coherent attack Barberini as an obscurantist since what one inevitably wants to do is somehow assert that Galileo was brilliantly right to assert that the Earth really does move without losing points by incidentally violating the modern taboo against treating any scientific result as absolutely true. I guess you could claim that we believe that the sun is really in the middle, but we only believe so provisionally. As Thony points out, though, it was far from clear in 1620 whether the sun was in the middle or not.

    • isomorphismes

      So I’ve never understood this in reading casually about ancient Greek philosophers. Aristotle talks about God with a capital G but didn’t the Greeks have, like, a pantheon? You know, Hera, Athena, Zeus, ‘n’at?

    • isomorphismes

      Pope Urban wasn’t exactly channeling Karl Popper when he chopped logic with Galileo, but his epistemology is not so far from 20th or 21st Century commonplaces about the status of empirical knowledge.

      How so?

  11. Michael Fugate

    Had the church proved that geocentricity was true?

  12. Anger Bear

    @ Michael Fugate: Does the term “default position” sound familiar?

  13. Jeb

    What motivated Galileo and Foscarini to move to the position they did? I am speculating that it was a massive political miscalculation.

    I don’t know anything about this period but my instinct is to want to understand the social life operating in this system, the political networks, patronage system and how academic rivals sought to manipulate the rules of the gane to their advantage?

    The politics of this period is not exactly stable. Building networks and making political calculations must have been hedged with difficulty in such a fluid, changing environment for everyone involved.

  14. Ken

    Thony C – a rather inaccurate comment to claim as you do:
    “The Church never declared the heliocentric theory to be false they only said that it had not been proved”

    In 1616 they did actually rule on this. Eg:
    “by order of His Holiness and the Most Eminent and Most Reverend Lord Cardinals of this Supreme and Universal Inquisition, the Assessor Theologians assessed the two propositions of the sun’s stability and the earth’s motion, as follows: That the sun is the center of the world and motionless is a proposition which is philosophically absurd and false, and formally heretical, for being explicitly contrary to Holy Scripture; That the earth is neither the center of the world nor motionless but moves even with diurnal motion is philosophically equally absurd and false, and theologically at least erroneous in the Faith.”

    And Galileo was warned that if he:
    “did not acquiesce in this injunction, you should be imprisoned.” He was “ given an injunction by the then Father Commissary of the Holy Office in the presence of a notary and witnesses to the effect that you must completely abandon the said false opinion, and that in the future you could neither hold, nor defend, nor teach it in any way whatever, either orally or in writing; having promised to obey, you were dismissed. Furthermore, in order to do away completely with such a pernicious doctrine, and not let it creep any longer to the great detriment of Catholic truth, the Holy Congregation of the Index issued a decree which prohibited books treating of such a doctrine, and which declared it false and wholly contrary to the divine and Holy Scripture.”

    • Ye Olde Statistician

      a proposition which is philosophically absurd and false

      “Philosophically false” meant that it was false in science. That is, the scientific evidence against it was overwhelming, as the next post demonstrates. Because the science seemed so absolutely firm, they accepted that certain descriptions were literal. In a very ironic way, the Church erred in this case insofar as they deferred to and agreed with the science.

      • Ken

        So you accept Thony C was wrong to claim:
        “The Church never declared the heliocentric theory to be false.”?

        The evidence us surely clear. No interpretation required.

      • Ye Olde Statistician

        So you accept Thony C was wrong to claim:
        “The Church never declared the heliocentric theory to be false.”?
        The evidence us surely clear. No interpretation required.

        It is unclear what meaning you intend. When the matter came up, the Church referred the science to a committee of scientists, which made the determination. That is, it was the scientists who declared heliocentrism to be not only false, but absurd. You may as well say that a court of law “declares” DNA analysis to be valid simply because it has accepted expert testimony on the matter. Or that the Dover court “declared” ID to be false.

  15. Ken

    It’s not my meaning – it’s Thony Cs and I think this is absolutely clear. And your hesitancy is telling.

    Your argument of the science bring firm is strange considering the situation of outstanding scientists disagreeing.

    Perhaps you should name the “scientists” who made the determination. Names and affiliations please. Did they consider any evidence for geocentricity and why should they dismiss the findings of well respected scientists line Copernicus, Kepler and Galileo?

    • Tim O'Neill

      Hear hear! We should then determine if any of these “scientists” were in the pay of the Templeton Foundation. No, let’s not bother. Let’s just assume that.

      • Ken

        Yes, and I suspect the commission did not include that outstanding astronomer Galileo Galielei – even though he was producing the most important empirical data about the solar system at the time.

  16. Ken

    Ye Olde Statistician – I must take you to task for being misleading in your claim:

    “the Church referred the science to a committee of scientists, which made the determination. That is, it was the scientists who declared heliocentrism to be not only false, but absurd. “

    Now I know you cannot name these “scientists.” They were theologians.

    Finocchiaro’s “The essential Galileo” describes it as such:

    “24 February: A committee of eleven consultants reports to the Roman Inquisition their unanimous opinion that the heliocentric and heliostatic thesis is philosophically absurd and formally heretical; and that the geokinetic thesis is philosophically absurd and theologically erroneous.”

    Sobel’s “Galileo’s Daughter” says:

    ““In February 1616 “the cardinals of the Holy Office framed the Copernican arguments as two propositions to be voted on by a panel of eleven theologians:”

    And the preamble to Galileo’s “Inquisition’s Sentence (22 June 1633)” the cardinals (actually only seven out of the ten cardinals in the commission) say:

    ““the Assessor Theologians assessed the two propositions of the sun’s stability and the earth’s motion, as follows:
    That the sun is the center of the world and motionless is a proposition which is philosophically absurd and false, and formally heretical, for being explicitly contrary to Holy Scripture;
    That the earth is neither the center of the world nor motionless but moves even with diurnal motion is philosophically equally absurd and false, and theologically at least erroneous in the Faith.”

    The higlighting of the word theologian is mine.

    Its amazing how many myths are being promotied by apologists around galileo these days. It is particularly disingenious to claim that the church and iqisition supported the science and Galileo and others opposed the science.

    Perhaps “disingenious” is too mild.

    • Ye Olde Statistician

      You keep missing the part where they say philosophically absurd. Natural philosophy was the term used to refer to the natural science of the day. The panel reported on the consensus science of the day.

      Don’t forget, heliocentrism had been falsified in good Popperian fashion by the absence of the predicted stellar parallax and the lack of eastward deflection in falling bodies. (Good evidence why Popper is inadequate.) The latter was not empirically settled until the 1790s when Gugliemini dropped weights down the inside of the spiral staircase of the tower of the Univ. of Bologna. The former was apparently settled when Calandrelli observed parallax in a-Lyrae. (“Apparently” because some have contended that he could not have actually observed it. However, it was reported in the literature in 1803.)

      My old history prof used to say that the most difficult part of understanding history was to study the past as present in the past. “We must consider Salamis as if the Persians might still win.” So to understand 1610 (as opposed to its polemic use as myth), you must forget anything that happened in 1611 et seq. As the physicist Duhem said, Galileo was right for all the wrong reasons, while Osiander and the others were wrong for all the right reasons. But guessing right is not science.

      It is particularly disingenious to claim that the church and iqisition supported the science and Galileo and others opposed the science.

      Don’t blame us; blame Huxley, “Darwin’s Bulldog,” who for the reasons Thony has outlined said that “the Church had the better case.” The rule was that a particular interpretation should be dropped when empirical evidence showed definitely that it was wrong. Bellarmine wanted to wait for the empirical evidence; Galileo wanted to be taken on faith.

      • Ken

        You meaqn where you theologians made the claim?

        Heliocentrism had not been falsified – and Popper had not yet been born. Both Kepler and Galileo were clear that they could not observe paralax because of the distance of the fixed starts.

        You guys continue to ignore what then inquisition and the Chruch put in black and white – their objection to Copernican theory because it violated their scriptural interpretation. And that they were moving against that theory becuase of its growing acceptance.

        This hegomony of theology had to be overturned before science could beocme intdependent and the scientific revolution occur (see Galileo’s revolutionary contribution).

        If Galieleo “want to be taken on faith” he would have become a theologian and noit worried about empirical evidence at all.

  17. Pingback: Galileo’s revolutionary contribution | Open Parachute

  18. Pingback: Galileo’s revolutionary contribution | Secular News Daily

  19. Ye Olde Statistician

    This is a bit lengthy and may impose on ThonyC’s forbearance. Therefore, it is a final word.

    You meaqn where you theologians made the claim?

    I am not a theologian, but a statistician, with some background in physics and history. Thony, so far as I can tell, is an atheist. It is only that he does not let his religious skepticism cloud his understanding of history.

    Heliocentrism had not been falsified – and Popper had not yet been born.

    Popper popularized the use of modus tollens. This logical tool had been around since the time of Aristotle and was well-known to the ancients, Arabs, and medievals.

    Both Kepler and Galileo were clear that they could not observe paralax because of the distance of the fixed starts.

    You cannot save one unproven hypothesis by throwing in a second unproven hypothesis. Aristotle, Archimedes, Ptolemy, and the rest had calculated the distance to the fixed stars using their brightness and apparent diameters and had found them an unimaginable 73 million miles away. Even so, parallax should have been evident. Of course, the scientists were wrong, but you can hardly blame everyone else for accepting their conclusions.

    You guys continue to ignore what then inquisition and the Chruch put in black and white – their objection to Copernican theory because it violated their scriptural interpretation.

    The summulae convicts Galileo of “having held and believed a doctrine which is false and contrary to the divine and Holy Scripture.” Note that they first describe it as “false,” and it was because they had been reading scripture in the light of that long-established science that they believed the new hypothesis to be contrary to it.

    This hegomony [!] of theology had to be overturned before science could beocme intdependent and the scientific revolution occur (see Galileo’s revolutionary contribution).

    The site linked to is a tendentious one and ought not be taken literally. It contained several errors of historical fact as well as of interpretation.

    If Galieleo “want to be taken on faith” he would have become a theologian and noit worried about empirical evidence at all.

    Galileo declared that the comets of 1618 were emanations in the earth’s atmosphere; whereas the Jesuit astronomers claimed they were bodies in highly eccentric orbits that had come from somewhere beyond the moon. In the Dialogo, Galileo had claimed the ocean tides were sloshing caused by the earth’s rotation and so conclusive proof. But centuries before, Aquinas had written that the tides were caused somehow by the moon. GG also set the Copernican model against the Ptolemaic model, even though the Ptolemaic model had by then been conclusively falsified and replaced by Tycho Brahe’s model, which he did not mention. (The Tychonic model delivered predictions as accurate as the Copernican. And the Copernican model still contained twenty or so epicycles, don’t forget. It was not computationally simpler.) GG also completely ignored Kepler, who actually did come up with a correct model.

    Don’t forget, a geocentric model of some sort was obvious to the senses. You could see the stars going around the earth. This conclusion was firmly established well before there was a Church by such folks as Aristotle, Archimedes, Ptolemy, et al., and was accepted as true by the ancients, the muslims, and the Christians. (It was not accepted by the Chinese, who still believed the earth was a flat blanket with China in the middle. They had nothing like logic or Euclidean geometry.) For a 14th century critique, one which puts forth the principle of relativity, see: http://www.clas.ufl.edu/users/ufhatch/HIS-SCI-STUDY-GUIDE/0040_nicoleOresme.html Note that Oresme addresses the scriptural objections late in the game and dispenses with them. He was qualified to do so because – unlike Thony and the others here – Oresme was a trained theologian as well as a natural philosopher.

    • You’re welcome to continue commenting on this thread as long and as often as you wish

    • Ken

      Sorry, “you theologians” should have read “your theologians.” I was not referring to you or Thony but to the panel of eleven theologians you had described as scientists. An obvious dig at your fibbing to support a preconceived claim.

      The blog I referred to is mine. It quotes from the documents of the time. I would be pleased for you to point out any errors of fact or interoretation you claim in discussion on the blog. Obviously I don’t take your blanket dissing seriously.

      Your attempt to claim that Tycho’s model has replaced the Ptolemaic model is clearly shown as another fib by the documents. The Church was concerned only with protecting the Ptolemaic model against the Ceprican one. They do not discuss Tycho’s model.

      Galileo was cleary wrong anout the tides (however I found his discussion in the Two World Systems very ingenious for the times). Great scientists are very often wrong. It is rather desperate for modern day critics to have to fall back on that little story.

    • isomorphismes

      (It was not accepted by the Chinese, who still believed the earth was a flat blanket with China in the middle. They had nothing like logic or Euclidean geometry.)

      Great topic for another post! I didn’t get my satisfaction here http://ninjaruski.tumblr.com/post/15538493348/a-response-to-isomorphismes-question-2-2 and i’m sure someone in this community knows better.

  20. Ye Olde Statistician

    Sigh.

    I was not referring to you or Thony but to the panel of eleven theologians you had described as scientists. An obvious dig at your fibbing to support a preconceived claim.

    A panel that concludes that a proposition is false in natural philosophy is simply restating the consensus science of the day. Much as a judge in Dover rules that ID is not properly science. That does not make it a fundamentally legal decision. The judge was simply restating the consensus of the scientists.

    I would be pleased for you to point out any errors of fact or interoretation…. Obviously I don’t take your blanket dissing seriously.

    No, you have a pre-formed attitude and so read everything in the light of your metaphysic. It might be useful to take a skeptical and objective POV. History is always particular and local, but Late Moderns seem enchanted with grand Theories of History in which real people in real circumstances are reduced to cardboard stereotypes performing in morality plays.

    1. You quoted from the summulae that Galileo was convicted of “having held and believed a doctrine which is false and contrary to the divine and Holy Scripture” while eliding over the conjunction. The doctrine is first described as “false.”
    2. You referred to the hermetic mystic, Bruno, who was burned for heresy and implied that Galileo might have been subjected to the same fate. But anyone who has read up on the case knows that not only was this not in the cards, it was against the rules of the Roman Inquisition.
    3. You ignored the fact the geocentrism was accepted science long before the Christian church was ever in being, and was accepted as fact by people who rejected the Jewish and Christian scriptures.
    4. You state that “the church had proclaimed geocentricism a ‘fact’ because it (in their opinion) is revealed by ‘Holy Scripture’” when it was Aristotle, Archimedes, Ptolemy, and others who had no interest in Christian scripture who had proclaimed it a fact.

    The authors of the scriptures accepted the science of their day and used it in their imagery. But that the fact of geocentrism seemed so solidly affirmed led them to suppose that the literal meanings of these passages were likewise solidly affirmed. (After all, Science says so!) Then along comes Galileo telling them how to re-interpret the scriptures based on little more than his say-so and in the face of unanswered objections. Now, the Jesuits were half-way convinced. Grienberger was reported to have said that Scheiner was on board and even Clavius was not far from the Copernican system. Then GG had to get into a flame war with Grassi over the comets of 1618, where Grassi had made meticulous telescopic observations and was clearly right on the science while GG had not even bothered to observe them, and so the Jesuits, who had earlier celebrated Galileo, sat on their hands when he went on trial. Grienberger supposedly said that if Galileo had only known how to retain the friendship of the Jesuits, he could have written anything he wanted about the motions of the earth.

    Your attempt to claim that Tycho’s model has replaced the Ptolemaic model is clearly shown as another fib by the documents. The Church was concerned only with protecting the Ptolemaic model against the Ceprican one. They do not discuss Tycho’s model.

    Look again. The church objected to the dual motion of the earth, which was held as absurd in philosophy and doubtful in philosophy. (A rotating earth seemed more acceptable than a revolving earth, probably because the lack of parallax was more obvious a falsifier than the lack of eastward deflection.) The Tychonic model also had a stationary earth, and so was also compatible with the text.

    Tycho’s model had replaced the Ptolemaic model among astronomers, including the Jesuit astronomers. The Ursine model was geocentric like Tycho’s but had a rotating earth, which was considered doubtful but not absurd. See the cover of Riccioli’s Almagestum novum (1651), where the Tychonic and Copernican systems weigh their pros and cons in a balance pan, while Ptolemy lies prostrate on the ground, feebly claiming that he will rise again. Riccioli’s book is the one that the Pope commissioned Galileo to write, weighing the best arguments in favor of and against each of the two main theories. https://thonyc.wordpress.com/2010/11/12/galileo%E2%80%99s-great-bluff-and-part-of-the-reason-why-kuhn-is-wrong/

    Galileo was cleary wrong anout the tides (however I found his discussion in the Two World Systems very ingenious for the times).

    As I said, it was long known that the tides were due in some way to the moon. Aquinas noted it 400 years before Galileo, and even Galileo’s friends tried to tell him it was a bad argument. Not only that, but it contradicted Galileo’s own response to the Objection of the Winds. Oresme had made better arguments 250 years earlier, and Galileo must have known Oresme’s work because he cribbed from it (e.g., the proof of the mean speed theorem).

    A crucial point little known among Late Moderns is that up until this time, astronomy was not considered part of physics, but a specialized sort of mathematics. The job title for an astronomer was “mathematicus.” An astronomical model had the sole purpose of making accurate predictions of celestial events, and in this the Tychonic model was just as accurate as the Copernican model. IOW, both models “fit the data” (or “saved the appearances,” as they said back then). But even Ptolemaic astronomers did not claim their epicycles were physically real. (This put Copernicus in a hard place. He clearly believed his model was physically real; but he had as many epicycles as Ptolemy.) A large part of the scientific revolution was the shift of astronomy from the math department to the physics department. The telescope was the primary instrument for doing so, because the telescope for the first time enabled astronomers to discover new physical facts much as Columbus had discovered a new world. This is very nicely described in Toby Huff’s new book Intellectual Curiosity and the Scientific Revolution.

  21. Ken

    Ye Olde Statistician – science is much easier – one can refer to reality and be accepted by opponents. Reality seems to be a thing you avoid.

    Judge Jones’ decison was not a scientific one – it was a legal one. But that’s not the point. Anyone who claims today that Jones was a scientist to give his decision authority wouild be lying. Just as you were, or perhaps just mistaken, when you claimed that the panel of assessor theologians were “a committee of scientists, which made the determination. That is, it was the scientists who declared heliocentrism to be not only false, but absurd.“

    The decision of that panel had a theological status, not a scientific one.

    Where is your integrity. Surely you should acknowledge the mistake, if it was a mistake, and not try diverting attention away from your actual claim.

    Point 1: My quote of the sentence did not avoid or omit the conjunction at all. It was an accurate quote. Another misrepresentation?

    Point 2; Whatever the rules (and lets face it Galileo was threatened with torture which was somewhat against the rules for his age and prominence) victims were surely going to see a threat in the talk of proceedings. And Bruno was recent history. With your apparent mistaking of a panel of theologians for scientists I am hardly going to take your opinion of what “anyone who has read up on the case knows”.

    Point 3: I ignored nothing of the sort – the article was not about the ancient Greeks – it was about Galileo’s contribution to the scientific revolution.

    re Tycho’s model: I have not yet seen any reference to this in the Church’s documents. Purely reference to the Ptolemaic model. But I am always ready to see new evidence. However, your interpretation alone will not be suffcient. Where specifically do they refer to Tycho’s model?

    You should read Galileo’s Two World Systems – he discusses the effect of the moon at length. It is silly for people to drag up the moon – the problem was the lack of understanding of gravity.

    As I said in the other thread I am quite shocked at the motivated approach commenters like you seem to have. You are willing to convert theologians into scientists, give a judge the status of a scientist, etc., as well as misrepresent my article. And you are arrogant enough to declare Galileo’s astonmical observations and comments on verification against reality as “based on little more that say-so in the face of unanswered objections.”

    What did Galileo ever do to you to make you want to misrepresent him in this way?

    What was that about a pre-formed attitude?

    • Ken you keep claiming that the panel who investigated Galileo’s letters to Castelli and Christina were theologians and that their judgement was a purely religious one and not a scientific one. In all aspects of these claims you err. As you have a tendency to either ignore or to deny everything that I say I shall first of all quote from a source that you have already, justifiably, praised. In his The Essential Galileo, Finocchiaro, writes:

      An investigation was launched that lasted about a year. As part of this enquiry, a committee of Inquisition consultants reported that the earth’s motion is absurd and false as a matter of natural philosophy and heretical, or at least erroneous, as a matter of religion and theology.

      Finocchiaro clearly states that the judgement was both scientific and religious. Also the committee is referred to as consultants not as theologians so why do you insist categorically that they were theologians. I know that you have a personal dislike of the Catholic Church but I think even you must admit that the Church was probably intelligent enough to consult natural philosophers and astronomers when considering questions of cosmology and astronomy.

      You also seem to be of the opinion that a theologian is naturally ignorant on matters of cosmology and astronomy. In the 17th century this is far from true. Roberto Bellermino, universally acknowledged as the greatest theologian of the age and the man who actually, at the Pope’s request, imparted the committee’s findings to Galileo, had himself lectured on astronomy as a young teacher at the University of Louvain. The leading Catholic astronomers of the century in Italy Clavius, Grienberger (both good friends of Galileo), Kircher, Riccioli and Grimaldi all had doctorates in theology as they were all full members of the Jesuit order. Other non Jesuit Catholic theologians also possessed a solid grounding in astronomy. A strict division between the two disciplines that you appear to believe in did not exist in the 17th century.

      • Ken

        Thony, my objection was to the panel being described as a committee of scientists. The church itself described it as a panel of theologians. So that claim was just not true. It was a lie.

        A simple check will show you that I am correct in this. Check out the primary documents. Look at the consultants report. 

        If you read the Inquisition’s sentence (given in Finocchiaro’s “The Essential Galileo”)  you will see it’s description as a panel of theologians. The consultants actual report (given in Finocchiaro’s “The Galileo Affair”) lists them by name.

        There has been some discussion over punctuation in the sentence which could be used to argue that the whole consideration was only theological. But that’s beside the point. It was a group of theologians making the assessment – and it is out of step with the fact that a number of astronomers of the time would have disagreed with their assessment on the science.

        My conclusions on this are derived completely from the primary sources. They do not arise from any dislike of the church you imagine. That would be a very naive approach on my part and I reject your insinuations on this. They are unwarranted.

        You are also unjustified in attributing to me an opinion of the complete ignorance of all theologians of the time on astronomy. That is not my position. I am aware that some had these skills. In fact some of these preferred the heliocentric model. They were no doubt distressed by the foolish position their church adopted in prohibiting Galileo and others from holding that opinion.

        I do not believe, as you claim, that there was a strict division between astronomy and theology at the time. Another misrepresentation on your part.

        My point has always been, though, that this break between theology and science which has certainly occurred since was a necessary part of the scientific revolution.

        I wish people would actually take the trouble to read what I write and resist the temptation to simply consult their preconceived prejudices about me.

      • Ken

        Responding to your claim “the Church was probably intelligent enough to consult natural philosophers and astronomers.”

        I have not yet found any evidence the panel of theologians consulted anyone else. They do not mention doing so in their report. This is very brief and I could post it here if you wish. Or maybe do another post on this subject at Open Parachute. It would be worth people having access to the facts seeing they have been distorted.

      • isomorphismes

        Ken, I’m reading this discussion after the fact and haven’t picked a side to believe in as someone new to the history. But when you say

        Thony, my objection was to the panel being described as a committee of scientists. The church itself described it as a panel of theologians. So that claim was just not true. It was a lie.

        you have neglected what Thony just said, which is:

        …the judgement was both scientific and religious…[many examples of overlap between theology and astronomy]… A strict division between the two disciplines that you appear to believe in did not exist in the 17th century.

        You didn’t controvert Thony’s claim that theologians may also be scientists. Either you have to do that, or accept the label of “scientist” (or natural philosopher, whatever) notwithstanding their theological pedigrees.

      • Ken:
        You indulge in the cheap tricks of a third class confidence trickster. You present only a part of the picture deliberately omit important information and hope that the victim will interpret the offered information incorrectly in your interest. Just to present the assessments of the assessors and their Church titles and ranks without explaining what these thing actually mean within the Renaissance context is a cheap meaningless trick.

        “…this proposition is foolish and absurd in philosophy…“ The assessment is dealing with cosmological questions, which within the Mediaeval/Renaissance divisions of knowledge is a part of philosophy and not astronomy. This statement can be loosely translated into modern terms as that the proposition that the sun is at the centre of the world and devoid of motion is scientifically unsound. Guess what, based on the empirical evidence and the scientific theories of the early seventeenth century this judgement is fully correct as is the similar judgement concerning the proposition about the earth. In this the assessors are thus completely correct in their judgements something of which Galileo was very much aware. This is the reason why Galileo’s Dialogo is strong on polemic but very weak on scientific arguments. He knew that he couldn’t even come close to disproving geocentrism or proving heliocentrism so he bluffed.

        I assume that your listing of the Church titles of ranks of the assessors is supposed to demonstrate that they are not qualified to make the scientific judgements that I have just sketched but in that you would be mistaken, as I have already explained to you in the past; an explanation that you simply ignore because it doesn’t fit your agenda. I will however explain it again for the benefit of isomorphismes.

        All of the assessors are high-ranking Church officials so they all almost certainly have doctorates. Most of them probably had doctorates in theology but one or the other probably had a doctorate in law or even a double doctorate in law and theology. This means that they are all intelligent well-educated men. Having gone through they Renaissance university system all of them will have as undergraduates in the mediaeval university liberal arts system had a solid grounding both in mathematical astronomy and cosmology, as already mentioned part of the philosophy curriculum. Can we say the same for Galileo? What was his doctorate in? Oh I forgot he was a dropout medical student he didn’t even have an undergraduate degree. Also without further investigation it is impossible to tell if any of the assessors have deeper interests or knowledge in astronomy or cosmology. Ken would want you to believe that this is unlikely but is it?

        The Papal commission that researched the Gregorian calendar reform just thirty years earlier was led by a cardinal, a bishop who later became a cardinal and a patriarch of the Orthodox Church all of them theologians, so no scientific knowledge or? All three were highly competent in astronomical theory that’s why they were chosen for the job. More examples? Christoph Clavius was one of Galileo’s mathematical sponsors as he was a young, unknown mathematician looking for a job. He also supplied Galileo with the necessary education in mathematics and physics by sending him transcripts of his own lectures. A mathematician? Yes, but also a Jesuit monk with a doctorate in theology. Galileo went on to make some important scientific discoveries but a discovery is only scientific if it can be replicated and confirmed by others. Galileo’s telescopic discoveries were confirmed by Lembo and Grienberger both excellent astronomers but also Jesuit monks with doctorates in theology. Galileo’s laws of fall were confirmed by Grimaldi and Riccioli both excellent scholars who made multiple contributions to the evolution of science in the seventeenth century but both Jesuit monks with doctorates in theology. I could go on but I think I have made my point.

        Without proper historical investigation of the individual it is impossible to know the level of scientific knowledge possessed by an individual at the beginning of the seventeenth century. With many of them it is not even possible then. Your attempt, Ken, to imply that the assessors were scientifically ignorant is very bad history and typical of the blind polemic that you propagate.

      • Ken

        Tony C – you have misrepresented me as my reply below should indicate. I had being showing the claim that the consultant theologians was a panel of scientists was incorrect.

        I reject your justification for the consultant theologians conclusion. It was clearly giving the answer the inquisition wanted – and purely on authority without any philosophical/scientific justification. (And as I point out the inquisition could no doubt have cobbled together a panel of natural philosophers to produce the same conclusions).

        The geocentric model was moribund requiring such deep ad hoc additions and the heliocentric model was getting important mathematical and astronomical supporting evidence. Galileo made important contributions here.

        I wonder at your motives to accuse me of being a “third class confidence trickster” for simply providing the full text of the consultant report, unamended. And for attributing to me positions I do not have. Here I have simply been trying to correct some incorrect representations of the Galileo affair which unfortunately seem to be creeping in to some hisorical accounts. If my arguments are faulty please deal with that – with facts – not by personal attack.

    • The only reference to Ptolemaeus in the Church documents translated in Finocchiarro’s The Essential Galileo is in Galileo’s Fourth Disposition and refers to Ptolemaeus’ opinion that the earth stands still and the sun moves. Here Ptolemaeus is named because in the offending document, for which Galileo was on trial, his Dialogo he only refers to the systems of Copernicus and Ptolemaeus. This alone is very strange, and a question of great interest to historians of science, because he completely ignores both the most scientifically advanced system, Kepler’s, and the most widely accepted, Tycho’s.

      The Ptolemaeic system had, at least in its pure form, been effectively refuted by the telescopic observations of Galileo, amongst others, of the phases of Venus in 1611 – 1613. This meant that those who wished to retain a geocentric and geostatic system were forced to adopt a Tychonic or semi-Tychonic one. The Church finally accepted a full Tychonic system around 1620, which remained the official Catholic doctrine till well into the 18th century, although nearly all Catholic astronomers accepted Kepler’s elliptical heliocentric system much earlier.

      • Ken

        Thony C – you claim “The Church finally accepted a full Tychonic system around 1620, which remained the official Catholic doctrine till well into the 18th century”

        Could you provide me with a reference, a primary source, for this? It should have been mentioned in the sentence if adoption occurred in 1620 – I don’t recall seeing it.

        I agree that Kepler and Tycho get little mention in The Two World Systems and think that may have been professional jealousy and ego on Galileo’s part. But his discussion of the inner planets there in effect means he did cover some of Tycho’s ideas. On the other hand even Kepler refused to support Tycho’s model despite pressure to do do. So I am a little suspicious about current claims for it’s support.

        However, I will await your references.

      • Isomorphismes,
        Here is the full text of the theologians report and their names and affiliations:

        “Assessment made at the Holy Office, Rome, Wednesday, 24 February 1616, in the presence of the Father Theologians signed below.

        Propositions to be assessed:

        (1) The sun is the center of the world and completely devoid of local motion.

        Assessment: All said that this proposition is foolish and absurd in philosophy, and formally heretical since it explicitly contradicts in many places the sense of Holy Scripture, according to the literal meaning of the words and according to the common interpretation and understanding of the Holy Fathers and the doctors of theology.

        (2) The earth is not the center of the world, nor motionless, but it moves as a whole and also with diurnal motion.

        Assessment: All said that this proposition receives the same judgment in philosophy and that in regard to theological truth it is at least erroneous in faith.

        Petrus Lombardus, Archbishop of Armagh.
        Fra Hyacintus Petronius, Master of the Sacred Apostolic Palace.
        Fra Raphael Riphoz, Master of Theology and Vicar-General of the Dominican Order.
        Fra Michelangelo Segizzi, Master of Sacred Theology and Commissary of the Holy Office.
        Fra Hieronimus de Casalimaiori, Consultant to the Holy Office.
        Fra Thomas de Lemos.
        Fra Gregorius Nunnius Coronel.
        Benedictus Justinianus, Society of Jesus.
        Father Raphael Rastellius, Clerk Regular, Doctor of Theology.
        Father Michael of Naples, of the Cassinese Congregation.
        Fra Iacobus Tintus, assistant of the Most Reverend Father Commissary of the Holy Office.”

        Yes, of course they were asked to report on the theology and philosophy of the issue. And of course in those days (even in our day) one could be both a theologian and astronomer or mathematician.

        Their overconfident conclusions and sweeping claims of course ignore the reality of the actual situation. Their brevity probably indicates what was expected of them.

      • isomorphismes

        KenKen, thank you for providing the rich and relevant source material. It seems you are using the consultants’ religious affiliations and rank as evidence against their scientific/philosophical credibility. Am I wrong in saying this?

    • isomorphismes

      science is much easier – one can refer to reality and be accepted by opponents

      You haven’t been following the climate-change debate!

      • Oh, but I have. And I conclude that much of the opposition to the scientific consensus is politically or ideologically motivated. Within the science community there are degrees of difference (normal for complex subjects) which now and then get resolved by testing and validating against reality.

      • isomorphismes

        Just a joke. (Maybe not a funny one.)

      • Ken

        Yes, isomorphismes, you are wrong. I am not saying that. This discussion arose because one of the commenters claimed the consultant theologians was actually a panel of scientists. Clearly that was wrong.

        I imagine the church could have cobbled together a panel of mathematicians and astronomers, natural philosophers, to also give them the answer they wanted.

        The panel, as Finocchiarrio suggests, was hardly giving a proper in-depth assessment. To suggest that heliocentricism was “foolish and absurd” scientifically (philosophically) may have been the desired conclusion but it was hardly an honest one.

      • isomorphismes

        one of the commenters claimed the consultant theologians was actually a panel of scientists. Clearly that was wrong.

        Why clearly? Do we have some information about these men that they were not scientists/natural philosophers?

      • Ken

        Isomorphsmes, that commenter was Ye Old Statistician – you will need to put that question to him/her as they were making the claim that the consultant theologians were a committee of scientists. The documents clearly describe them as father theologians, not scientists. And in fact that term (scientists) would not have been used then. Clearly wrong to do so now.

        I myself have not yet found evidence that the consultants made any thorough investigation of recent scientific findings. Huge omission seeing Galileo was responsible for some of these.

        Their conclusions themselves seem simply based on their own authority and I really can’t see how they were justified to describe these scientific ideas the way they did (“foolish and absurd”). They were clearly not that even at the time (unless you interpret “foolish” in political terms related one’s safety.

      • isomorphismes

        I’m not sure on whom the burden of proof lies — on YOS to show the consultants were in fact philosophically expert, or on you to show they weren’t. But does that matter? This seems to be a key sticking point in the discussion which could be resolved easily with evidence.

      • Ken

        No it doesn’t matter and it’s not a sticking point – provided you don’t take the consultant theologians report as gospel. Just look at what was known at the time, the mathematical models and the new astronomical findings. Given that it seems to me that a group which called consideration of helicentricism “foolish and absurd” really discredits itself, doesn’t it?

        Perhaps unproven (neither was geocentricism) but not “foolish
        and absurd.”

        I mean if you are going to sit back and speculatively pontificate on the credibility of that report you will not make any progress. There is just so much else to consider.

    • Ken you keep going on about “evidence”. Would you be so kind as to detail the evidence accepted by Galileo which in your opinion should have led people in 1615 to abandon geocentrism a well established and strongly empirically supported theory.

      • Ken

        Geocentricism as a model required a large number of arbitrary ad hoc additions for it to work – a sign of it not according with reality, although enabling instrumental use.This would not have been lost on intelligent thinkers of the time and Galileo was not the only expert starting to reject the old model. In reality the geocentric model was not “strongly empirically supported” – hence the necessity of the arbitrary ad hoc adjustments to make it work.

        For modern commentators to suggest that the geocentric model was so good requires them to mistakenly ignore the need for arbitrary, unexplained, ad hoc, adjustments to produce the success they claim. They are not thinking scientifically.

        But I suggest you refer to Galileo’s writings of the time both for his arguments and his empirical evidence. The later ranged from his observations of the moon, Jupiter’s moons, the phases of Venus, sunspots, the estimation of star sizes and hence distance, etc. All things which undermined the old model.

        Given the shonkey nature of the geocentric model, the success of the Copernican mathematics and the existence of such new astronomical findings it was hardly honest for the consultant theologians to reject new ideas as philosophically “foolish and absurd” – but obviously that is what the inquisition required.

      • Ken, as usual when confronted with a question on the history of science, a subject of which you obviously know very little and understand even less, your answer consists of a lot of hand waving and a minimal amount of dubious or inaccurate facts.

        Which aspects of the geocentric model are ad hoc? Instead of parroting Alan Chalmers, who is by the way a fairly poor historian of science to put it charitably, make specific statements. Give factual descriptions to back up your claim. No need I will do your work for you. All geocentric models, and yes there are more than one, have supposedly ad hoc mathematical models to explain retrograde motion, whereas heliocentric astronomy does not need to explain it as it is an illusion caused by the fact that the system is heliocentric and the planet of the observer overtakes, in its orbit, another planet which thereby appears to display retrograde motion. However these mathematical devices are not ad hoc in the sense of Chalmers because they are not intended to be explanatory. This is by the way only one single “ad hoc add on” and not “a large number” as you claim and there are no others. Mathematical astronomical systems were, in antiquity and in the Renaissance, were not explanatory but predictive. Their function is to predict the daily position of planets and the occurrences of conjunctions, eclipses etc. for use by astrologers, cartographers, navigators etc. The worth of a system was judged on its utility and accuracy for these purposes. Mathematical astronomy should save appearances not explain them. Explanation was the province of cosmology a branch of philosophy. Heliocentrism triumphed over geocentrism not because it was true but because Kepler’s system provided the most accurate planetary tables. Original interest in Copernicus’ model was not cosmological but mathematical, could it produce more accurate planetary tables than Ptolemaeus’ model? The interest waned rapidly as it became clear that because it was based on the same database the answer was no. When discussing historical models it is necessary to know what the contemporary assessment criteria for assessment were and not try as you are very obviously doing to apply modern ones.

        Let us now examine your supposed evidence for the superiority of Copernicus over Ptolemaeus. I find it amusing that you tell me to read Galileo, as I have already done so in detail whereas I very strongly suspect that you have not. Let us now examine in detail the pitiful handful of supporting evidence that you actually mention by name.

        Galileo’s observations of the moon have no significance for the choice of system. They are a refutation of the Aristotelian theory of the immutability of the supraluna sphere but play no role as to whether the solar system is heliocentric or geocentric. The same goes for the sunspots. The moons of Jupiter merely disprove the Aristotelian dictum that everything in the heavens has the earth as its centre of rotation. Again it offers no proof or disproof for either system. Galileo’s observations of star discs, which are completely erroneous due to an optical illusion produced by the use of telescopes, are in fact a proof of geocentrism and not heliocentrism. I direct your attention to the relevant academic papers on the subject by Professor Christopher Graney. I sure that somebody as well informed as you claim to be will have no trouble finding them. Of course the failure of Galileo and everybody else to observe any stellar parallax was a very serious problem for the heliocentric theory and one of the central facts that very much slowed down its acceptance. However we’ve been here before and you Ken being omniscient refuse to accept this fact known to and accepted by all historians of astronomy because it doesn’t fit the fantasy that you have created in your own head. I’m not quite sure what you intend with the statement “Copernicus’ mathematics”. Copernicus had produced a mathematical model of a heliocentric system, so what? It was actually more complex and required more circles than the then current geocentric system from Peuerbach. They both used the same deferent epicycle system. The only major difference was that Copernicus had eliminated the equant point. Do you know what the first thing was that Kepler did in his system? He put the equant point back in, so much for Nicky’s only mathematical innovation. In terms of its predictive power for planetary movement, as I’ve already said, it was no better than the geocentric system in use at the time and in some cases even worse. So what is the supposed importance of Copernicus’ mathematics that you wish to point to here?

        This brings us finally to the one and only correct claim that you have made, the phases of Venus. These do indeed prove that Venus cannot be in orbit around the earth but must be in orbit around the sun. This does not however disprove a geocentric model or prove a heliocentric one, as Galileo well knew. The phases of Venus are fully conform with a so-called Heracleidian model in which Mercury and Venus orbit the Sun which in turn orbits the Earth; a system known since antiquity and very popular in the Middle Ages. The phases are also fully conform with the so-called Tychonic geoheliocentric systems in which the planets orbit the Sun which in turn orbits the Earth; systems that dominated European astronomy, and not only in Catholic countries, between about 1620 and 1660 when the Keplerian heliocentric system finally became accepted. The partial evidence for heliocentricity in the phases of Venus was more than cancelled by the very real and very serious physical problems produced by the concept of a moving Earth.

        Having demolished all of your supposed evidence for a necessary acceptance of heliocentricity in 1616 we will turn to what Galileo himself did when he was lobbying in Rome for the Copernican system in 1615/16. Fully aware of the problems involved in any of the supposed pieces of evidence you suggest Galileo didn’t use any of them. So what did he use? He used his notorious theory of the tides. Why notorious? Because it is scientifically total crap and even contradicts the available empirical facts. Contrary to you claims there wasn’t any convincing evidence available for heliocentricity in 1616 apart from the work of Kepler and as everybody knows Galileo rejected that by ignoring it.

      • Ken

        Bloody hell, Thony C., Galileo may have been an argumentative old bugger but he had nothing on you. “Playing the man rather than the ball” has always seemed to me a sign of weakness, (or a substitute for factual material) – so I won’t respond in kind. Let me, though, just make some specific responses:
        1: Alan Chalmers – where the hell did he come from? And what specifically have you got against him?
        I am not aware of Chalmers having any pretentious to being a historian. He is a philosopher of science. In my judgement one of the better ones.
        I was going to say I was unaware of anything he had written on history (I have only read a few of his books) but on reflection obviously a good philosopher of science must also know a fair bit of the history of science. And in his book “The scientist’s atom and the philosopher’s stone,” which I have read and reviewed, he does use that history in a general sense. However, no one, least of all me, is going to use that book as a source for scientific history. Philosophy – that’s a different matter – I find the book very enlightening in that respect.
        Personally, I prefer to go to Maurice Finocchiaro on the history of the Galileo affair. But I wouldn’t go there for philosophy of science, no way.
        Two things I like about Finocchiaro is that:

        a: he provides ready access to some of the primary documents (not all but a lot more than most), and
        b: he has specialised in the controversy that has existed around the Galileo affair. He details this in his book “Retrying Galileo, 1633-1992.” That interests me because that controversy, often driven by ideological agendas, exists still today – obviously.

        Incidentally, as well as philosophers of science knowing something of science history I think that historians of science should know more about the philosophy and epistemology of science. I have seen a huge failing in that respect where some contributors have even claimed that heliocentricism was “falsified” by things like the inability to detect stellar parallax. That is absolute rubbish, as a little bit of epistemological reflection would make obvious. (Incidentally, the inability to observe stellar parallax was also a problem [on a smaller scale] for the geocentric model and its true understanding requires consideration of stellar sizes and distance which critics usually ignore).
        Finally, on Alan Chalmers – the only reference I can find to Galileo in the book I mention is this:

        “The stationary earth theory correctly predicts that a stone dropped from a tower will land at its foot. But once Galileo had shown that this would also be the case for a steadily spinning earth, the experiment could not count as evidence for either a stationary or steadily spinning earth. . . . theories, if they have sufficient leeway, can be made compatible with evidence by means of suitable adjustments. if we are free to pick the circular orbits corresponding to the cycles and epicycles in Ptolemy’s astronomy so they fit observations of planetary positions then that fit, is not genuine evidence for the theory.”

        Now I wish some of the commenters here could draw the philosophical lesson given there.
        2: Thony C. – you claimed that geocentricism was a ” well established and strongly empirically supported theory.” Now you claim the opposite – you have thrown empirical support out the window and instead assert The worth of a system was judged on its utility and accuracy for . . . . astrologers, cartographers, navigators etc.” It is of course true that the practical utility of a model is the criteria for many users – but not for those wishing to understand reality. One has only to read Galileo’s letters and articles to see that the truth of a model, or its degree of correspondence to reality, was important to him, and I assume to many others at the time. It seems to have been of interest to the church and inquisition.
        So of course astrologers and navigators may not have been interested in the gerrymandering required to make the geocentric model work for them but the obvious fact that these were arbitrary, ad hoc, unsupported adjustments would not have been lost, and obviously weren’t, on deeper thinkers.
        It’s not a matter of me “obviously” making the mistake of applying modern criteria – it’s a matter of you choosing just one criteria that was important at the time and ignoring the criteria advance by Galileo, Kepler and others. An important mistake as herein lies a problem that had to be overcome for science to win its freedom from religious philosophy – a freedom that was necessary for the scientific revolution.
        3: My reference previously to some of the contributions made by Galileo using telescopic observations were not meant to claim that any one specific observation “falsified” geocentricism – just that they all added to its demise (give that it’s empirical basis was become more and more unsteady). I can understand why you want to “explain away” each observation (as did Galileo’s opponents at the time) – and you strangely want to protect the geocentric model by removing a requirement for an empirical basis. You obviously have your reasons for that.
        However, I take you up just on your rejection of my point about “the estimation of star sizes and hence distance.” I think Galileo made important arguments and advances here (even regarding naked eye observations) which critics always seem to ignore. However, you claim that Galileo’s observation were “due to an optical illusion produced by the use of telescopes.” Galileo argued that naked eye observation could not give reliable estimates of size (or brightness) and distance because of an optical illusion. There’s a conflict there – I can understand Galileo’s argument – I can’t understand yours.
        Now I know that some of Galileo’s opponents claimed that telescopic observations were unreliable (many of these refused to look through a telescope). But given that telescopic use only increased and has produced some amazing knowledge what is you evidence that their bitching was anything more than an example of personal rivalry?
        Galileo’s introduction of the telescope to astronomy lead to important jumps in human knowledge. He is rightly respected for that. But it is an obvious case of how instruments, computers, (Haak calls them “helps”) aid in our understanding of reality. Of course there are inevitable problems as we learn how to make them and use them – but let’s not throw out the baby with the bathwater. Or use such teething problems to reject what have clearly been important leaps in our understanding of the universe. I can understand the emotions of Galileo’s rivals at the time, but what are the motives of today’s critics who fall back on those sort of arguments?

      • “Geocentricism as a model required a large number of arbitrary ad hoc additions for it to work”
        Isn’t the assumption that the earth moves without us noticing it also an example for an arbitrary ad hoc addition?

      • “Incidentally, the inability to observe stellar parallax was also a problem [on a smaller scale] for the geocentric model and its true understanding requires consideration of stellar sizes and distance which critics usually ignore.”

        Not sure I can follow you here. Didn’t Tycho and Riccioli reject heliocentrism largely on the grounds that they inflated stellar sizes? (See Graney’s articles mentioned by Thony above)

      • Ken

        Baerista, I don’t see any “assumption” about the movement of the earth, rather that was a conclusion from empirical evidence supporting heliocentricism. That is very different to the ad hoc adjustments required to get the model to fit the data – these had no empirical support. They were simply fudge factors adjusted to fit the data. It’s important to understand the difference otherwise you end up calling everything an ad hoc adjustment. 

        Because of lack of observable stellar parallax a heliocentric model required the fixed stars to be much further away than was believed previously. Galileo used brightness observed telescopically to argue that was the case and that the apparent brightness observed with the naked eye was an optical illusion. The stars could be a lot closer with a geocentric model because parallax was observed across the diameter of the earth rather than the earth’s orbit. However the fact that no parallax was observed indicated the stars were still further away than some people believed even with geocentricism.

      • “…rather that was a conclusion from empirical evidence supporting heliocentricism.”
        After your last exchange with Thony, I’m not sure you’ve yet managed to convincingly outline what this empirical evidence was. Your claims about stellar size and Galileo’s stance on it are completely refuted by recent research. I suggest you read:

        Christopher M. Graney, “On the Accuracy of Galileo’s Observations,” Baltic Astronomy 16 (2007): 443-49.

        —— “Objects In Telescope Are Farther Than They Appear: How diffraction tricked Galileo into mismeasuring the distances to the stars,” The Physics Teacher 47 (2009): 362-65.

        —— “Seeds of a Tychonic Revolution: Telescopic Observations of the Stars by Galileo Galilei and Simon Marius,” Physics in Perspective 12 (2010): 4-24.

        Christopher M. Graney and Timothy P. Grayson, “On the Telescopic Disks of Stars: A Review and Analysis of Stellar Observation from the Early Seventeenth trought the Middle Nineteenth Centuries,” Annals of Science 68 (2011): 351–73.

        Christopher M. Graney, “Science Rather Than God: Riccioli’s Review of the Case for and Against the Copernican Hypothesis,” Journal for the History of Astronomy 43 (2012): 215–26.

        —— “Riccioli Measures the Stars: Observations of the Telesopic Disks of the Stars as Evidence against Copernicus and Galileo in the middle of the 17th century.”

        Click to access 1004.4034.pdf

        —— “Regarding how Tycho Brahe Noted the Absurdity of the Copernican Theory Regarding the Bigness of Stars, while the Copernicans Appealed to God to Answer that Absurdity.” http://arxiv.org/ftp/arxiv/papers/1112/1112.1988.pdf

      • MikeFlynn

        The Copernican model actually had more circles than the then-current edition of Ptolemy. (Peuerbach’s?) The Moon was on an unprecedented double-epicycle, and Mercury librated arbitrarily across its epicycle. Since the orbit of each planet was solved as a separate problem, they did not orbit a common center. (Nor did they have a common plane.) The Sun was offset from the center of the model (just as the Earth was offset from the center in Ptolemy’s model). The center of the model was the geometric center of the Earth’s circular orbit rather than the Sun itself. The solution of Mars was so badly off that the position was incorrect by more than 10 arc-minutes, iirc. He introduced a third motion of the Earth which Tycho later showed was due to observational error.
        The Copernican system needed a great deal of ad-hoc adjustment to work. That was because astronomy was in those days a specialized branch of mathematics and was distinct from physics and cosmology. It’s purpose was to “save the appearances” so the model was jiggered to match the data. Which really is much like what Late Modern computer modelers do.
        + + +
        The parallax problem was real. You can’t really save one unproven hypothesis by tossing another in another unproven hypothesis. The physicists had good reasons for supposing the stars to lie just beyond Saturn. Even with telescopes, the stars appeared to be discs. (The small aperture of 17th cent. telescopes resulted in aberration of the image.) And given these apparent diameters and the apparent brightness of the stars vis a vis Saturn or Jupiter, the stars were either closer than 100d(Saturn) or were absurdly huge. At that distance, parallax =should= have been visible. Its lack =required= a stationary Earth. Hence, the physicists regarded the revolution of the Earth as “foolish and absurd.” The board of consultators simply accepted the judgment of the “settled science.” (Neither judge nor jury in the Dover case were evolutionary biologists; but they didn’t have to be.)
        The rotation of the Earth was marginally more plausible, but the absence of observable Coriolis effects argued that it too was absurd.
        + + +
        In the Dialogo, Galileo referred to his opponents as “dumb idiots” and “mental pygmies, hardly deserving the name of human beings.” That might pass for Renaissance wit, but even in the Renaissance it was not quite scientific proof.

      • Ken

        Baerista, we had been discussing what empirical evidence at the time (1615) could have been used to support an alternative heliocentric model. Galileo’s observation of star brightness via telescope and description of illusion involved in naked eye observation were obviously useful empirical factors. Particularly as the showed arguments against hrliocentrucusm based on non-observation of parallax effects to be groundless.

        You have quoted Graney’s papers – apparently in an attempt to discredit Galileo’s argument. But you don’t explain how they do? Would you care to do so?

        The little I have read of Graney suggested the effect he considered (diffraction) would have caused underestimation of distance estimates based on apparent size. (Galileo also used brightness). So Graney may be correct the estimates of star size by astronomers at the time were too large. But how does this negate Galileo’s arguments on non-observation of the parallax effect? Surely they just strengthen the argument.

        Humanity has been underestimating the distance to the stars, size if the galaxy and universe through all of history.

        In the abstract of one of Graney’s papers he concludes:

        ” Galileo’s methods and observations were good, but since he was unknowingly observing diffraction artifacts and not the physical bodies of stars he greatly underestimated the distances to the stars.”

      • Ken

        MikeFlynn, true – “Neither judge nor jury in the Dover case were evolutionary biologists.” Especially as their was no jury.

        But look at the huge difference. We have Judge Jones’ excellent summary of evidence plus transcripts of expert witnesses for and against the school board.  A reader is sbke to get a good idea of the science and politics inolved at the time.

        We have nothing like that from the consultant theologians. Their pronouncement may be suitable for those prepared to take such pronouncements as authoritative but they tell you nothing of the actual empirical evidence or state if debate at the time.

        The consultant theologians report is more like a declaration of faith in intelligent design from the Discovery Institute (their wedge document for example) than Judge Jones’ 137 page summary

        I really suspect the objectivity, and investigative skills, of anyone relying on this pronouncement to tell us what the “settled science” was at the time. The fact that we are discussing treatment of one of the most outstanding astronomers of the time because he argued against geocentricism is surely a clear indication that the science was far from settled. (Bloody hell -I hate the term “settled science.” It usually indicates an agenda because science is never settled)

        I think some people are confusing an instrumental model, gerrymandered to fit the data using fudge factors, with science. As Thony himself admits this model had important practical use but it did not describe reality. Yet some people are willing to give it more warrant than a more realistic model that fitted the data less well because it hadn’t been gerrymandered.

        I think your other point about parallax has already been answered.

        But my question is why go to this effort. What’s the motive for “taking sides” from this distance and uncritically rehearsing the old arguments, already dealt with by Galileo, in the process?

        It’s interesting to speculate.

      • MikeFlynn

        What’s the motive for “taking sides” from this distance and uncritically rehearsing the old arguments, already dealt with by Galileo, in the process?

        Historical accuracy is generally to be favored over fable. There is a rolling horizon in human historical consciousness which, in literate societies, falls at about three centuries before the present. Thus, Arturius dux milites of the fifth century became King Arthur and a Round Table by the eighth century. And Charlemagne of the eighth century became the Charlemagne of the chansons de geste in the eleventh century.

        So, too, for us, the 17th century in the popular imagination has slipped over the horizon, passed out of historical consciousness and into myth. Accounts are shortened, complexities sloughed off, analogous figures fused, traditions “abraded into anecdotes.” Real people become culture heroes: archetypical beings performing iconic deeds. So Jamestown is reduced to Pocahontas and John Smith while Massachusetts becomes Plymouth Rock and the First Thanksgiving. And the convoluted details and scientific problems associated with the transition from geostatic to heliostatic math models has been simplified to the archetypal culture hero Galileo performing the iconic deeds that validate our Modern way of life.

      • Ken

        Sure, MikeFlynn. But why rehearse one side of the argument? In my mind this is what is creating myths rather than preventing them. Sure, Galileo is considered a scientific hero – because of his great contribution. These day in science we have such heroes but do not make them idols. I read quite a few biographies of the great scientists and what impresses me is that none of them these days a hagiographies. The human side of these individuals is not ignored – we are given a fuller picture -warts and all. This is great because it does reveal that science is a human process and scientists are just as human as the rest.

        I think that the science community has a pretty good handle on the objective picture of Galileo. We celebrate him for his astronomical and mechanical discoveries. His persecution by the church actually gets very little interest, far less than I think justified but that, I think, reflects the average scientist’s lack if interest in philosophy. That is, I believe we need to do more to celebrate Galileo’s philosophical contributions than we do.

        In contrast I find here an unbalanced preoccupation with Galileo’s trial and a bias towards the church. That I think is promoting some myths, particularly on the Internet. 

        So here’s an important question for you: you claim “In the Dialogo, Galileo referred to his opponents as “dumb idiots” and “mental pygmies”. . .”. Now I know Galileo was a cantankerous old bugger but in the interests of respect for history and avoiding a mythology could you provide a reference – to the day at least. I ask because I don’t remember him using those terms (no problem -my memory is hardly perfect) and I cannot find either of those four words from a quick digital search if my copy. Of course that could be differences in translation .

        Now I am not casting doubt. Just want to read the words and the context myself. I am sure you can appreciate that.

      • Ken, since you are apparently not going to engage with Graney’s research yourself, I will try to summarize it as succinctly as I can:

        In 1617, Galileo made unpublished observations of the double star Mizar. He determined their distance from the Earth by assuming that they were roughly the same size as the sun. Due to diffraction, their measured distance ended up so short as to necessitate clearly observable annual stellar parallax, the expected angles being in the range 11.5 arc-minutes in the case of Mizar A and 7.6 arc-minutes for Mizar B. Neither of them did. Parallax, albeit minimal, should still have been detectable if both stars had been assumed larger than the sun by an order of magnitude.
        By the modern standards you like to invoke, this alone should have made Galileo more skeptical towards his own theory than he was, because the empirical data supported a Tychonic system (albeit with diurnal rotation of the Earth). This conclusion had already been drawn by Simon Marius in 1614, based on the same kind of observational data that Galileo had at his hands. What did Galileo do instead? He published the Diaologo 15 years later, in which he STILL claimed that the stars were all like the sun and offered calculations of their distance from the Earth that would have necessitated observable parallax. He even had his mouthpiece Salviati explicitly outline what should be observable given a double star like Mizar – but in doing so he obscured the fact that he already had made such an observation in 1617 and that it did not match his prediction. Riccioli later called him out on exactly this point. Interestingly, his theory of the tides shows a similar pattern of ignoring empirical data that did not fit his theory. …

        Not quite the same as what you are saying – or am I misreading you?

      • Ken

        No, Baerista, I really don’t want to further invest my limited time on a list of Graney’s papers because I can’t see the relevance. Graney concluded (I repeat):
        “ Galileo’s methods and observations were good, but since he was unknowingly observing diffraction artifacts and not the physical bodies of stars he greatly underestimated the distances to the stars.”

        We have been underestimating these distances through all of history (which underlines Douglas Adams’ description of the universe as really, really large).  And Galileo’s estimates were of course infinitely better than those required by geocentric models. Graney is not arguing that Galileo was incorrect in arguing that stellar distances were much greater than currently believed at the time.

        The claims made by commenters here are that (correctly) the non-observance of orbital parallax was a problem for the Copernician model with the then current understanding of stellar distances, and

        Galileo was making an unwarranted assumption of much larger stellar distances, or introducing an ad hoc adjustment without empirical evidence for it (incorrect).

        I have pointed to Galileo’s work describing the optical illusion of naked eye observation leading to overestimates of star and planet brilliance, size and closeness. Together with his telescopic observations which reduced such illusions and therefore enabled a more realistic estimate.

        This empirical evidence was used by Galileo, and presumably others, to describe why at that time no orbital parallax were observed, even with telescopes.

        I really can’t see the relevance of citing unpublished observation you claim made by Galileo. Perhaps you are suggesting that Galileo was dishonest, ignoring his own data because the conflicted with his model. (Do I see another Galileo myth in the making?). You refer specifically to “the Diaologo” where the relevant quote is (I think):

        “Let us look at the Dog Star which is very beautiful and larger than any other fixed star, and which appears to the naked eye not much smaller than Jupiter, when we remove it’s head of hair in the manner indicated, it’s disk will be seen to be so small that it will be judged one twentieth that if Jupiter: indeed whoever lacks perfect vision will have great difficulty perceiving it . .

        Here Salviati is describing the optical illusion where naked eye observation leads to overestimates of brightness, size and closeness. Hardly a reason to get your knickers in a twist about an unpublished (and therefore uncheckable) estimate made previously. Similarly I can’t see the relevance of your reference to Galileo’s mistaken arguments from tidal effects. Unless it is to somehow play the man rather than the ball – to promote a story of Galileo somehow being dishonest with evidence).

      • You keep missing the point in spectacular fashion: Galileo underestimated the distances to such a degree that parallax should have been measurable ACCORDING TO HIS OWN DATA AND ADMISSION, if the geokinetic hypothesis was correct. Salviati explicitly says so in the Dialogue, stating, inter alia, that double stars would “appear in court to give witness to such motion in favor of the earth.” What he conceals is that he had already made this test in 1617, but without the desired result. As even the passage you quote indicates, the later Galileo saw stars not as dimensionless points, but as “disks” and these disks (also known as Airy disks) were still large enough to create the mentioned problems, even if viewed through a telescope. Your account of what Galileo did is simply no longer tenable and Graney’s papers show precisely how and why. The claim that Galileo was dishonest is the straightforward conclusion from this recent research. It’s not my problem that you’re such a fanboy that you won’t read anything that might dispel your cherished notions of Galileo’s intellectual immaculacy.

  22. Pingback: Historical fiction | Open Parachute

  23. Pingback: Weekly Picks « Mathblogging.org — the Blog

  24. Ken

    I have now put up the full text 0f the assessor theologians we have been discussing in my blog article Historical fiction.

    Comments welcome.

  25. Pingback: Galileo’s modern critics | Open Parachute

  26. Pingback: Galileo’s modern critics | Secular News Daily

  27. Pingback: Galileo’s revolutionary contribution | Secular News Daily

  28. Pingback: Historical fiction: "The Sky's Dark Labyrinth"

  29. isomorphismes

    The heliocentric hypothesis says that heliocentricity offers a possible model to explain the observed motion of the planets; it says nothing about the truth-value of this model. The heliocentric theory says that the universe is in reality heliocentric.

    So the heliocentric theory is in fact wrong. The universe does not centre on the Sun, nor even on the galactic central point.

  30. Pingback: At the local viewing of the transit of Venus, I... • see things differently

  31. Ken

    Baerist, there is a simple solution:

    You claim evidence for scientific fraud by Galileo “ACCORDING TO HIS OWN DATA AND ADMISSION.”

    Well, provide a reference link to this. I want to see what he actually “admitted” and what data he gathered. This is the normal process and I would think the required process for such a serious allegation. I can’t consider such things just on anyine’s personal assurance.

    Otherwise you are asking me to march in line with a particular groupthink and do thus by calling names (fanboy). If your argument has any validity you can do far better than this.

    Meanwhile you are diverting away from my point that the problem of non observance of parallax was well explained – no ad hoc fudge factor was required.

  32. Ken

    Thanks for the reference Baerista – unfortunately it is to one of Graney’s papers not Galileo’s work. Nevertheless I will have a read of it.
    Clearly you are basing your whole argument on Graney alone, not on Galileo – do you know of any other astronomer who agrees with Graney on these points? I ask because you seem to think this one person destroys the complete argument about the effect of distance on parallax and its non-observance at that time. You might (emotively) think my comment on the non-observance is a “false claim” but you haven’t shown it. And of course we know now that it is completely correct. It does explain why parallax was not observed at the time.
    So I will get back to you with my assessment.
    Meanwhile a comment on disks. You seem to argue that my quote from “the Dialogo” was basically a calculation of distance based on apparent disk size. It wasn’t and it’s not clear that Galileo was suggesting anything definite about the 1/20 of Jupiter’s disk (he suggested some people with less than perfect vision would not even see the star). He was after all just describing the optical illusions when naked eye observation was used.
    I have noticed that in parts Galileo is using “size,” or even “disk,” more as an estimate of brightness than actual diameter. That is always how I have understood his argument. So I am interested to see any example of him using estimated diameter for seriously estimating distance of a star (as he did with planets). (Yes, Graney may have something taken from observation notes but I would like to see a serious argument rather than a back of the envelope (or notebook) calculation done speculatively – we all do that).
    It’s interesting because Galileo does seem to differentiate between the “disks” of planets and the “disks” of stars. He comments on the sharpness of the planetary disks compared with stars. In fact he seems to take the attitude that the telescope only went partway to reducing the optical illusion with naked eye observation. For example in discussing the planets, and their phases, he comments “Mercury’s disk is so small and its light so bright (due to its being so close to the sun) that the power of the telescope is not enough to shave its hair and make it appear completely shorn.”
    So, at this stage if Graney has based his discussion purely on notes then this is a case of making a mountain out of a molehill. Bugger me, I hope I don’t live to see the day when my critics will be digging into my lab notes, or spread sheets, and attempting to build a story out of my speculative early calculations to then claim I was guilty of scientific fraud because I didn’t include such speculations in my published papers! Scepticism and criticism is important in science but it should be done in good faith.
    As I said, I will get back to you when I have read Graney’s paper.

  33. Ken

    Here are some thoughts after reading Graney’s paper.

    Firstly I do not see any evidence that Graney thought Galileo guilty of fraud, dishonesty or deception as you claim. So I assume it is your conclusion alone. You assert that it “is the straightforward conclusion from this recent research.” But then you show your lack of objectivity by saying of me (just because I don’t agree with the fraud charge):  “you’re such a fanboy that you won’t read anything that might dispel your cherished notions of Galileo’s intellectual immaculacy.” You really have trouble with Galileo, don’t you?

    Graney specifically suggests Galileo had a commitment to heliocentricism because of its mathematical and logical sense which was not swayed by the apparent contrary evidence. I basically agree with that conclusion (and see it as a mark of good research on Galileo’s part) although I think Graney expresses this too harshly. Perhaps this is because Graney himself does not fully appreciate the nature of creative research.

    Graney does not appear to have a record of research as a natural scientist – more in teaching and investigation of historical research. Maybe that has lead him to give more significance to speculative observation notes than a more experienced scientist would.  In fact he should have picked this up from his reading of the Dialogue and the comments on this very aspect of Copernicus – to stick with reason despite the contrary information of his senses. In this case the lack of  phases for Venus (unobservable in Copernicus’s time) and the insufficient variation in apparent sizes of Venus and Mars. He does at least appear to be aware of the parallel with Copernicus, given he quotes Wilson on this.

    Incidentally, I see this question of parallax the same way – Galileo was unconcerned about its non-observation realizing that the distances involved made detection difficult. He would no doubt have like to measure it but we know now it was not possible within the limitations of his telescope. Perhaps Graney may have interpreted detection failure as a “falsification” but any mature scientist recognizes that as naive.

    You can see from fig 4 in Graney’s paper how dubious any measurement of distance based on what could be seen with telescopes of the day would have been. I can see why Galileo did not publish this and why any calculation he made (even his drawings) would be only speculative. I have done the same thing many times myself and as I say it would be bad faith for anyone to now use such speculation in my lab notes as evidence of scientific fraud.

    That figure also confirms what Galileo had said in comparing telescopic images of planets and stars. He used such observation to estimate sizes and distances of planets but I don’t see that he seriously used it for stars – except to argue they were much further away than estimated by the unaided eye.

    I have already commented that Galileo appeared to recognize that the telescope only went part way to removing the optical illusion of unaided vision. Elsewhere he also says;

    “The planets present their bodies perfectly delineated and round, . . . However, the fixed stars do not look to the naked eye bounded by a circular periphery, but rather like blazes of light, shooting out beams on all sides and very sparkling; and with a spyglass they appear if the same shape as when they are viewed by simply looking at them, . . .”

    Finally, I have been unable to actually find quotes you give and some of Graney’s in “The Dialogue” despite electronic searching.  I suspect in Graney’s case he is actually quoting from an editor’s introduction or preface rather than Galileo (or in some cases from earlier writings of Galileo which may have somehow got into his copy if the book). In your case (and in Mike’s case) perhaps secondary sources are also being quoted.

    (Perhaps that is why Mike has not got back to me).

    So I conclude that the arguments you and you mates (group thinkers perhaps?) have made relying on Graney’s work are another example of a Galileo myth in the making.

    I thank you for the opportunity to witness this and to check out the specific paper you base it on.

    • “Firstly I do not see any evidence that Graney thought Galileo guilty of fraud, dishonesty or deception as you claim. So I assume it is your conclusion alone.”

      Fraud and deception are your words. Dishonesty is mine. Based on what was known and observable at the time, Galileo’s attempt to pass over the problem in the Dialogo qualifies as dishonesty. Your denial of this is only psychologically interesting.

      “Graney specifically suggests Galileo had a commitment to heliocentricism because of its mathematical and logical sense which was not swayed by the apparent contrary evidence.”

      Contrary evidence is not swayed by prefacing it with the word “apparent”. Just another bit of presentistic, ex-post hand-waving on your part.

      “Graney does not appear to have a record of research as a natural scientist – more in teaching and investigation of historical research.”

      You mean the way you do not appear to have a record of research as a historian of science? Projecting, much?

      “Maybe that has lead him to give more significance to speculative observation notes than a more experienced scientist would.”

      After “apparent”, we get now “speculative” to make uncomfortable data go away. So we are to conclude that Marius and Riccioli were merely “speculating” when insisting on what was observable. Why? Because Ken says so. Because HE knows the earth goes round the sun and no pesky historian can take that away from him. Congratulations.

      “Incidentally, I see this question of parallax the same way – Galileo was unconcerned about its non-observation realizing that the distances involved made detection difficult.”

      I am happy for you that you’re so unconcerned about the fact that Galileo was unconcerned. And yet the distances implied by stellar size and acknowledged in the Dialogo gave other astronomers grounds enough for being concerned – in spite of all your hand-waving.

      “He would no doubt have like to measure it but we know now it was not possible within the limitations of his telescope.”

      The joys of presentism…

      “You can see from fig 4 in Graney’s paper how dubious any measurement of distance based on what could be seen with telescopes of the day would have been.”

      It’s dubious for you now, knowing what it is known now about diffraction. Recognize a pattern here?

      “That figure also confirms what Galileo had said in comparing telescopic images of planets and stars. He used such observation to estimate sizes and distances of planets but I don’t see that he seriously used it for stars – except to argue they were much further away than estimated by the unaided eye.”

      Of course he didn’t “seriously” use it for stars – it would’ve undermined his theory. And yet he does the calculation for us in the Dialogo, ignoring that stars 2160 times more distant than the sun should still exhibit parallax.

      “I have already commented that Galileo appeared to recognize that the telescope only went part way to removing the optical illusion of unaided vision.”

      In his Reply to Ingoli (1624) he “appears to recognize” quite the opposite. It sure is convenient to be able to put anything that goes against your theory down to optical illusion. No “arbitrary ad hoc assumptions” at work here, I’m sure.

      “Finally, I have been unable to actually find quotes you give and some of Graney’s in “The Dialogue” despite electronic searching.”

      Scholars have developed these devices called “footnotes” that direct you to the source reference and page in question. The juciest bit is pp. 382-83 in Drake’s translation. Look them up and cry yourself to sleep at night.

      “I suspect in Graney’s case he is actually quoting from an editor’s introduction or preface rather than Galileo (or in some cases from earlier writings of Galileo which may have somehow got into his copy if the book). In your case (and in Mike’s case) perhaps secondary sources are also being quoted.”

      That’s just another “arbitrary ad hoc assumption” on your part.

      “So I conclude that the arguments you and you mates (group thinkers perhaps?) have made relying on Graney’s work are another example of a Galileo myth in the making.”

      Once you’re done killing the other 500k+ Galileo fanboys out there, you can start complaining about “group thinkers”.

      “I thank you for the opportunity to witness this and to check out the specific paper you base it on.”

      You’re welcome.

  34. Ken

    I guess, Baerista, your obvious emotion in this issue makes continued discussion irrelevant and perhaps I shouldn’t ask this. However, being aware that you are using the Drake translation while I have Finocchiaro’s translation  (plus the apparent problem of quoting translator’s introductions and now footnotes) could you please provide the text from the Dialogue on which you base your claim that in that work Galileo specifically used apparent stellar disk size (not brilliance or magnitude) to estimate a stellar distance? Again I am not saying he didn’t, just that electronic searching of my copy, and rereading Day 3 twice yesterday, failed to find such evidence.

    But apart from that I really don’t see any more concern over the then inability to detect stellar parallax than the “problem” that we have had a very good, but incomplete, standard model in particle physics but were unable to detect the required Higg’s field until very recently (and as yet not confirmed publicly).

    That’s science for you – reliable but never certain.

    Meanwhile I agree with Finocciaro that the ongoing controversies over the “Galileo affair” are fascinating. Why should people today be taking sides, passionately and emotively dissing a historical figure and rehearsing old arguments against him while ignoring his replies to those arguments?Finocciaro sees one motive in pro or anti church sentiment. He also mentions desires for mystery (eg promoting Keppler while dissing Galileo) but I think there may well be more human psychology involved.

    • Are you telling me it didn’t occur to you that Finocchiaro’s book is subtitled “A New Abridged Translation and Guide” and that this might be the reason you can’t find the passage – and that instead you’re hypothesizing that Graney might have gotten through peer review masking a “translator’s introduction” as Galileo’s actual words? You are sounding more and more like a conspiracy theorist, especially when you keep making innuendos about the motivation of those who insist on a realistic understanding of history (granted, calling you a “fanboy” is not much different).
      Now let’s don our tin foil hats and look at the Italian original:

      http://it.wikisource.org/wiki/Dialogo_sopra_i_due_massimi_sistemi_del_mondo_tolemaico_e_copernicano/Giornata_terza

      First the piece I quoted from:
      Salviati: […] E se io con un piccol motto vi apersi la mente, voi con un altro fate sovvenire a me, non esser del tutto impossibile che qualche cosa in qualche tempo si trovasse osservabile tra le fisse, per la quale comprender si potesse in chi risegga l’annua conversione, talchè esse ancora, non men de i pianeti e del Sole stesso, volesser comparire in giudizio a render testimonianza di tal moto a favor della Terra: perch’io non credo che le stelle siano sparse in una sferica superficie, egualmente distanti da un centro, ma stimo che le loro lontananze da noi siano talmente varie, che alcune ve ne possano esser 2 e 3 volte più remote di alcune altre; talchè, quando si trovasse co ’l telescopio qualche piccolissima stella vicinissima ad alcuna delle maggiori, e che però quella fusse altissima, potrebbe accadere che qualche sensibil mutazione succedesse tra di loro, rispondente a quella de i pianeti superiori.

      Then the calculation:
      E prima, suppongo con l’istesso Copernico, e concordemente con gli avversarii, che il semidiametro dell’orbe magno, ch’è la distanza della Terra al Sole, contenga 1208 semidiametri di essa Terra; secondariamente pongo, con l’assenso de i medesimi e con la verità, il diametro apparente del Sole, nella sua mediocre distanza esser circa un mezo grado, cioè minuti primi 30, che sono 1800 secondi, cioè 108.000 terzi. E perché il diametro apparente d’una stella fissa della prima grandezza non è piú di 5 secondi, cioè 300 terzi, ed il diametro di una fissa della sesta grandezza 50 terzi (e qui è il massimo errore de gli avversarii del Copernico), adunque il diametro del Sole contiene il diametro d’una fissa della sesta grandezza 2160 volte; e però quando si ponesse, una fissa della sesta grandezza esser realmente eguale al Sole, e non maggiore, che è il medesimo che dire, quando si allontanasse il Sole tanto che il suo diametro si mostrasse una delle 2160 parti di quello che ci si mostra adesso, la distanza sua converrebbe esser 2160 volte maggiore di quello che è ora in effetto; che è quanto dire che la distanza delle fisse della sesta grandezza sia 2160 semidiametri dell’orbe magno. E perché la distanza del Sole dalla Terra contiene di comune assenso 1208 semidiametri di essa Terra, e la distanza delle fisse (come si è detto) 2160 semidiametri dell’orbe magno, adunque molto maggiore (cioè quasi il doppio) è il semidiametro della Terra in comparazione dell’orbe magno, che ’l semidiametro dell’orbe magno in relazione alla distanza della sfera stellata; e per ciò la diversità di aspetto nelle fisse, cagionata dal diametro dell’orbe magno, poco piú osservabile può esser di quella che si osserva nel Sole, derivante dal semidiametro della Terra.

      Your appeal to citation error having failed, the next phase in your defence will predictably be to wail about “context”, telling us what Galileo really meant, how it’s all just hypotheticals and how Graney is distorting his true intention. This is perfectly fine by me, but I’m done with this discussion.

  35. Ken

    It’s good you acknowledge that “fanboy” is inappropriate for this sort of discussion. 

    So it appears there may be differences in the two translations – one of my suggestions.  Perhaps I will download a Drake version if I can find one.

    But does the fact you resort to an Italian version mean that your point is not supported by the Drake translation either?

    Or are you just playing silly buggers?

    What about just providing the English language version as evidence for your claim?

    Your last paragraph is putting words in my mouth in a very misleading way. I did not write the things you claim. In itself that paragraph is as childish as “fanboy.”

    Yes I would like to know the context as well as the actual text (in English) of what Galileo wrote.  I wish to check for myself your claims as I don’t find your assurances, in themselves, reliable. 

    It’s a very simple request that you keep avoiding.

    Nothing to get emotional about – perfectly normal request.

    Why hide behind the Italian when apparently the Drake translation says the same thing?

    • Jesus Christ on a stick, I am amazed by this reply: how the heck can you “hide” behind the Italian, if that’s the original effing language and thus most faithfully represents what Galileo wrote???

      “But does the fact you resort to an Italian version mean that your point is not supported by the Drake translation either?”

      WTF? The Drake translation is the one quoted VERBATIM in Graney’s articles, for Christ’s sake, so it obviously supports the point. You just have to look up the articles I listed and you’ll get your translation. On its own, it does not seem to be easily available online.

      • Just to make sure you give me no more of this “silly buggers” crap. You wrote: “[…] could you please provide the text from the Dialogue on which you base your claim […].” Now I, like most historians, base my claims on original texts. Add to this the fact that the Italian original is extremely easily and reliably available on wikisource and you’ll get the reply you got from me. If you’re saying that you cannot read Italian, fine, but then you cannot adjudicate between translations anyway. If you’re saying that you wanted me to somehow give you the whole text of Drake’s translation, so you can check the context of Galileo’s words: I’m sorry mate, but there are limits to my generosity. The library’s the place for you in this case.

  36. “Yes I would like to know the context as well as the actual text (in English) of what Galileo wrote.”

    Galileo did not write in English.

    Quote of the day:

    If English was good enough for Jesus, it’s good enough for the schoolchildren of Texas.

    —Texas governor James Ferguson, 1917, explaining why he vetoed a bill funding the teaching of foreign languages in Texas schools.

    • Ken

      Yes Phillip, I had heard that story.

      Unfortunately I do not read Italian so the Baerista’s quote was of no use to me.

      However, I have found a copy of the Drake translation (google books), it does include the section referred to and I can see the context. So I have no need of any of  Baerista’s generosity.

      And this is certainly interesting. I can now see why he did hide behind the Italian and suspect even if I could read it the selection avoided the context. His anger was also a sign – I find unnecessary emotion and anger is often used to divert or avoid a straightforward investigation.

      Galileo was using the calculation to show where the critics of Copernicus were going wrong. (They argued that orbital parallax should have been easy to detect with heliocentricism.) It’s an example of a mistake which he then goes on to correct and show that such a calculation does not suggest that the orbital parallax was large enough to detect at the time.

      Baerista should have been very careful with his charge of dishonesty against Galileo – he is being dishonest himself and should have realized that there is no way he could prevent others from checking.

      It’s very early in NZ at the moment so I will get back in a few hours with the details -this really requires use of a PC.

      • You are being ridiculous, absolutely ridiculous. Do you really think I didn’t expect you to raise this pseudo-point? Why do you think I wrote above that “your appeal to citation error having failed, the next phase in your defence will predictably be to wail about ‘context’, telling us what Galileo really meant, how it’s all just hypotheticals and how Graney is distorting his true intention.”?
        So here we are. I also explicitly pointed out several times that the distances calculated by Galileo should still have made parallax detectable, as Graney shows in his papers, so your point is utterly vapid. Instead of carefully reading what Graney and I have written and reflecting on the data, you keep on arrogantly digging your own hole, showing that you’re a perfect exemplification of the Dunning-Kruger-effect. I’m done with you, you tool.

  37. Ken

    OK, I think this dispute with Baerista is resolved. My translation was indeed abridged, and now that I have access to the Drake version I have found the sections of text that Baerista referred to, and their context.
    Baerista has clearly been misrepresenting Galileo with his extracts and interpretations. Maybe acceptable in a blog discussion but I hope he hasn’t published these same accusations and biased sections.
    Some Background: The discussion has been about the problem that the non-observance of orbital parallax in the fixed stars was a problem for the Copernican model. My point was that Galileo had explained this was because the stars were actually far further away than most current astronomers believed. Being so the non-observance was not surprising.
    Baerista countered by claiming that Galileo had in fact made the calculations (based on apparent stellar sizes) and these showed that parallax should have been easily observable. That he had withheld that information in “The Dialogue” – proof that Galileo was dishonest. But strangely adding that he actually had included a calculation in The Dialogue showing that parallax should have been observed.
    Well he did include such a calculation, to illustrate the argument used by critics of the Copernican model, and went on to show their blunders – two major mistakes related to estimation of stellar sizes. Baerista was dishonest to claim these as Galileo’s own arguments and described them as follows:

    “Galileo underestimated the distances to such a degree that parallax should have been measurable ACCORDING TO HIS OWN DATA AND ADMISSION, if the geokinetic hypothesis was correct.”

    The relevant discussion in Drake’s translations starts about page 359 (unfortunately the few pages before that are not in the on-line version at google books). Galileo points out that Copernicus argued that orbital parallax:

    “remain imperceptible in the fixed stars because of their immense distance from us in comparison with the distance of Jupiter or of Saturn. Here the adversaries of this opinion rise up, and take what Copernicus has called ”imperceptible” as having been assumed by him to be really and absolutely non-existent.”

    Some of the commenters here have made the same unwarranted mistake of converting “imperceptible” to “non-existent.” Galileo goes on:

    “Now in order to reveal the folly of their entire method, I shall show that by assuming that a star of the sixth magnitude may be no larger than the sun, one may deduce by means of correct demonstrations that the distance of the fixed stars from us is sufficiently great to make quite imperceptible in them the annual movement of the earth which in turn causes such large and observable variations in the planets. Simultaneously I shall clearly expose to you a gigantic fallacy in the assumptions made by the adversaries of Copernicus.”

    He then goes on to do the calculation including this: ” the apparent diameter of a fixed star of the first magnitude is no more than 5 seconds, or 300 thirds, and the diameter of one of the sixth magnitude measures 50 thirds (and here is the greatest error of Copernicus adversaries), which appears to be in the Italian text quoted by Baerista.
    With those assumptions the distances of the fixed stars “would have to be 2,160 times” the distance of the earth from the sun. Which indicates that the orbital parallax of the fixed stars “would be little more noticeable than that which is observed in the sun due to the radius of the earth.” That is easily observable at the time (even without telescopes).
    These are not “admissions” of Galileo, he did not endorse these calculations (by critics of Copernicus). He goes on to say:

    “It is indeed wrong, since according to this author a star of the sixth magnitude would have to be as large as the earth’s orbit in order to justify the dictum of Copernicus. Yet assuming it to be equal only to the sun, which in turn is rather less than one ten-millionth of that orbit, makes the stellar sphere so large and distant that this alone is sufficient to remove this objection against Copernicus.”

    His brief calculations show that a star of sixth magnitude would have to be 10,648,000 times the size of the sun according to the critics of Copernicus.
    Discussion goes on to deal with the apparent size of the fixed stars as estimated by astronomers. He shows that most of them have overestimated size:

    “For they did not take care of the adventitious irradiation which deceptively makes the stars look a hundred or more times as large as they are when seen without haloes. “

    While telescopes helped remove this optical illusion he showed how even without telescopes a more realistic estimation of stellar size could be obtained. So he was quite critical of astronomers who made this mistake: (“Among these men are al-Fergani, al-Battani, Thabit ben Korah. and more recently Tycho. Clavius, and all the predecessors of our Academician.”).
    Galileo describes several ingenious methods for estimating stellar and planetary sizes with the naked eye which attempt to avoid the optical illusion. He then discusses other ways, using the planets, to estimate the distance of the fixed stars.
    Sorry this discussion may have been a bit complicated. But I think any objective reader of those pages in the Drake translation will see that Baerista has been misrepresenting Galileo. It is dishonest of him/her to therefore conclude that Galileo was the dishonest one. However, we can see how when one desperately searches texts for evidence to support one’s own prejudices it’s easy enough to find some.
    Unfortunately (and I repeat myself) this seems to happen a lot with Galileo. Even today commenters are emetoviely “taking sides” and supporting their arguments dishonestly.
    It is interesting to speculate on their motives.

    • MikeFlynn

      All that Ken has done is to claim that Galileo supported one unproven hypothesis by throwing in a second unproven hypothesis. He likely did, since Copernicus had raised the argument much earlier. The problem was that there was no proof of a vast stellar distance and Galileo was torqued off because people would not simply accept his just-so story at his word.

      So long as stars had perceptible disks, which they did in the small-aperture telescopes of that era, they would, by simple geometry, be the size of Earth’s orbit if they were much farther away than their brightness indicated. That the aberration was an artifact of the lens-grinding of the day is true, but it was not at the time proven. Science does not consist of lucky intuitions.

      An honest dialogue would have run:
      Sal.: There is no perceptible parallax because the stars are much farther away than you think.
      Sag.: That would do it.
      Sim.: But we know how far the stars are because of their relative brightness. And at that distance the parallax would be detectable if the earth moved. Besides, astronomers in their telescopic observations see the stars as small but definite disks.
      Sag.: He has a point.
      Sal.: Idiot. Those disks are optical illusions caused by optical aberration.
      Sag. & Sim.: Say what?
      Sal.: It’s something that hasn’t been discovered yet; but someday it will be.
      Sim.: Dang! I can prove anything I want if I can make up future data.
      Sag.: He has a point.
      Sal. (to Sag.): Remind me why there are three people in a dia-logue.
      Sim.: And what have you to say of Tycho and his system?
      Sal.: You mean his alleged observations and his “monkey planet” comets?
      Sag.: And what of Kepler and his elliptical orbits?
      Sal.: Kepler? Kepler who? Besides, the divine Copernicus wished to return to the pure Platonic circles by eliminating the equant, and did so.

  38. Ken

    OK Mike. I hope you don’t try to pass off that rubbish as a quote from the Dialogue.

    However, I repeat my last question to you:

    “So here’s an important question for you: you claim “In the Dialogo, Galileo referred to his opponents as “dumb idiots” and “mental pygmies”. . .”. Now I know Galileo was a cantankerous old bugger but in the interests of respect for history and avoiding a mythology could you provide a reference – to the day at least. I ask because I don’t remember him using those terms (no problem -my memory is hardly perfect) and I cannot find either of those four words from a quick digital search if my copy. Of course that could be differences in translation”

    Perhaps I should also search through the Drake translation.

    It’s amazing what you find when you check up on people’s claims by going to the sources.

    • MikeFlynn

      Well, one finds it all through the discussions of the affair; Koestler mentioned it, and it was cited by contemporaries; but you may be right. It may be that the Italian words can be translated differently. GG may have written it in his correspondence on in the marginal glosses, as he did in Grassi’s book. One certainly finds the ‘tude in his correspondence.

      Galileo marked up his copy of Grassi’s book: “piece of asininity,” “buffoon,” “evil poltroon,” “ungrateful,” and, addressing Grassi in The Assayer he wrote “You cannot help it… that it was granted to me alone to discover all the new phenomena in the sky and nothing to anybody else.” (So much for Tycho, Kepler, Harriot, Marius, Fabricius, Scheiner, and the rest.) He also referred to Tycho’s “alleged observations” and to comets as “Tycho’s monkey-planets.”

      It is quite clear how he regarded any perceived rival. Keep in mind that he called Grassi an evil poltroon (the Italian for which can also be rendered “idiot”) because Grassi had claimed comets ran on non-circular orbits from far beyond the moon (just as Tycho claimed) and had based this on meticulous observations of three comets. Galileo believed comets to be emanations in the Earth’s atmosphere; and had made no observations of any comets. The “ungrateful” part was evidently because Grassi had written about the heavens on which Galileo had exclusive right of discovery and had not mentioned Galileo’s name one single time.

      And, yes, the mock dialogue was satirical. One must explain such things I suppose to the tone-deaf fundamentalist.

  39. Ken

    Mike, it’s not a matter of I “may be right” but of you making a claim which you had no basis for (specifically about the “Dialogue.”)

    I guess it just shows the level of emotion here around Galileo. Hence the inability to understand his contribution to science – a contribution which was rightly celebrated by scientists in 2010, the International Year of Astronomy which marked the 400th anniversary of his first use of the telescope in astronomy.

    Why the emotion? Why must you insist in seeing scientists able to recognize such contributions as “fanboys” and “tone-deaf fundamentalists?” Why the need for such silly distortions of the facts?

    • MikeFlynn

      It isn’t a need, but a demythologizing of history. Although one may argue that replacing myth with more realistic assessments actually is a need. 19th century myth-making shortened accounts, sloughed off complexities, fused analogous figures, and abraded traditions into anecdotes. Complex real people became simplified culture heroes: archetypical beings performing iconic deeds. (See Vansina’s account of the transition from historical consciousness into mythic consciousness among pre-literate peoples. Among literate peoples it seems to take a bit longer.) Like all myth, the 19th century myths were intended to celebrate and validate the Way Things Are.

      Perfectly human people doing perfectly human activities provide little need to demonize anyone. Galileo’s contemporaries had valid empirical reasons given the data then available to doubt the Copernican system (and its many epicycles and inventions). They required more than additional assumptions to resolve. They required actual empirical observation. Galileo was simply unable to deliver such evidence. Had his Dialogue been honest, Salviati would have repeated Copernicus’ assertion that the parallax was simply too small to see and the apparent disks of stars would some day prove to be an artifact of the instrument. Simplicio would have responded that now Salviati was making up future data and that if the telescope were unreliable as regards the stellar disks in what else might it be unreliable, and Sagredo would have said we shall have to wait and see, because telescopes were being constantly improved and it was not unreasonable that they might one day show the stars as mere pinpoints. But Galileo took the approach that his unsupported claims should have been enough.

      Heck, Oresme had dealt with the Objection of the Arrow and the Objection of the Headwinds by postulating “common motion,” not only centuries before Galileo, but centuries before inertia was developed as a concept in physics. This was recognized as a possible answer to the objection, but it was also recognized that it was an answer that did not as yet have a justification.

  40. Ken

    Mike, go and read the section of the Dialogue I referred to. (You clearly haven’t already done so). You have an extremely biased version of the history and the science – are in the position of rehearsing the arguments of opponents of Copernicus and ignoring completely Galileos empirical evidence and arguments. Completely.

    Far for removing myths you are in the business of trying to recreate myths -against Galileo. A point that Finecchiaro brings out in his book Retrying Galileo, 1633-1992.

    One of Galileo’s great contributions was that he supplied evidence. He showed how the optical illusion had fooled previous astronomers and illustrated how even naked eye observation techniques could be used to reduce that overestimation of stellar sizes.

    The problem of diffraction and overestimation of stellar size with the telescope was of course there – but it is a red herring. Galileo underestimated stellar distances by a large amount (and so has almost every one else since) but the underestimations used to attack Copernicus were hundreds of times greater. You are using the present day perspective to pretend tis phenomena undermined galiloes measurements. They don’t – at least not in the direction you require. (If he had been aware of diffraction effects he could have argued that the fixed stars were even furhter away – but this was not required to explain why parallax had not been observed.)

    Today, scientists don’t mythologise the heroes of science. They general describe them warts and all in their biographies (even to a fault as I notice in my recent review of Stenger’s latest book). But both historians and scientists seem unwilling to apply the same realistic standards to those who persecuted Galileo. How often do we read of the conflicts within the Vatican and inquisitionh and how trhese (and the poliutcal situa5tions facing the church) influence Urban’s reaction to galieloe. Or even the personality faults of people like Urban? Hardly ever!

    So let’s stop this attempt at remythologising. Lets try to be objective and not take sides. Lets not continue hiding the scientific contributions Galileo made or the evidence he used to counter the critics of Copernicus.

    Maybe our difference are those inherent in the different perspectives of a scientist (who relies on reality to keep him honest, and tests ideas against reality) and an historian (who perhaps has less recourse to reality – and maybe ingorant of scientific mehtod).

    I hope not. But there seem to be some pretty big chips on shoulders around here when it comes to Galileo and science.

  41. Ken

    Sorry for the spelling – I have other things going on at the moment.

  42. Ken

    And for those wishing a more friendly to the reader version of Drakes translation go to the Trial of Galileo Home Page and click on Galileo’s Dialogue Concerning Two World Systems. The discussion here has been about extracts in Day 3.

  43. I did not intend to respond any longer, after realizing that you’re too full of yourself to ever recognize when you’re wrong, but your allegations and distortions are too egregious not to:

    All you talk about is pp. 359-60 in isolation, instead of reading it together with pp. 382-83, which is what I have been urging all along. Galileo’s “admission” is right there and makes all your bickering superfluous.
    You also keep ignoring that the only distances Galileo mentions anywhere in day three, REGARDLESS OF CONTEXT OR INTENTION, are consistent with measurable parallax of double stars, as described on pp. 382-83.
    Note that even you refer to him as saying that “While telescopes helped remove this optical illusion…” Fact is that Galileo did not clearly state that he considered the Airy disk as observed through the telescope to be an optical illusion and in facted stated the opposite in his reply to Ingoli of 1624, which undermines your entire argument.

    Reading recommendation:
    Christopher M. Graney: “Seeds of a Tychonic Revolution: Telescopic Observations of the Stars by Galileo Galilei and Simon Marius,” Physics in Perspective 12 (2010): 4-24.

  44. “Maybe our difference are those inherent in the different perspectives of a scientist (who relies on reality to keep him honest, and tests ideas against reality) and an historian (who perhaps has less recourse to reality – and maybe ingorant of scientific mehtod).”

    As an historian, I simply would like to congratulate you for this statement, which demonstrates to anyone who visits this thread what an utterly despicable, self-righteous and deluded person you are.

  45. Ian Wragg

    Sorry to interrupt the conversation. I am reading Kuhn’s “Copernican Revolution” and was wondering if anyone could recommend “The Eye of Heaven : Ptolemy, Copernicus, Kepler” by Owen Gingerich. I want to read a few books on the subject of the Copernican Revolution before I begin to concentrate on the individuals involved.

    • Gingerich’s book is a collection of his papers on the history of astronomy of varying, depth, approach and quality. Probably slightly dated now but definitely worth a look as Gingerich is truly one of the experts.

  46. Ian Wragg

    Thanks Thony, you could not recommend anything that includes Tycho in the story. I just get the impression that he is not included, going by the title of Gingerich’s book.

  47. isomorphismes

    So now that I’m having to admit that the ancients really WEREN’T that silly … I’m starting to wonder if other anti-historical prejudices I had were false as well. For example: “spontaneous generation” seems like a fairly daft idea of how small animals come into being. Is there a deeper story to be told that shows why that’s in fact a robust position?

    BTW, the point of this story (not its content) reminds me of the “Mpemba effect” http://math.ucr.edu/home/baez/physics/General/hot_water.html

  48. Pingback: Acceptance, rejection and indifference to heliocentricity before 1610. | The Renaissance Mathematicus

  49. Pingback: OPUS 500: A retrospective | The Renaissance Mathematicus

  50. Pingback: SHOCK! HORROR! OUTRAGE! RELIGION HINDERS PROGRESS OF SCIENCE! | The Renaissance Mathematicus

  51. Pingback: An interesting question. | Brain Tricks: Belief...

  52. Pingback: Galileo, the Church and Heliocentricity: A Rough Guide. | The Renaissance Mathematicus

  53. Ryan.Vilbig

    Good information. I’d never picked up on the hypothesis/theory distinction before. Makes sense though. Even now, scholastically trained philosophers will pause in a conversation to make a distinction between curiosity/studiosity.

Leave a reply to Tim O'Neill Cancel reply